Вы находитесь на странице: 1из 78

TROPMED 2013 6.

Kerja di salon poultry dan


ayamnya pada mati, ada gejala
TROPMED influenza, apa virus yg
mungkin nyerang?
1. Patient brought to clinic because of some
nodules on his arm and body and a. Orthomyxoviridae
sometimes feel pain with pressure. On
examinations we found erythematous 7. Zat apa yg digunakan utk membersihkan
nodules, smooth and shiny, diffuse feses burung?
infiltrate with ill define border, no
fluctuation or erosion. There were pain A. Iodin
on his elbow with nerve enlargement, B. Etanol 70%
best stain for examination? C. Etanol 90%
A. KOH
B. Giemsa 8. Best virus isolation utk kasus di atas?
C. Zn Stain A. Blood
D B. Nasopharyngeal aspirate
E C. Hand swab
D. Anal swab
2. Metode pengambilan darah untuk orang E. Nasal swab
infective endocarditis? 3 kali di lokasi
berbeda interval waktu 10 menit 9. Orang kena malaria, demam tinggi, kena
parasitemia 3,5%, kenapa terjadi kondisi spt
3. Orang kena malaria, dicek demam tinggi, ini? Merozoit dikeluarkan dari liver menuju
parasitemia +++, diberikan artemisinin, 3 darah
hari kemudian saat dicek masih ada
demam dan parasitemia ++++, keadaan 10. 45yo man was seen in the dermaotlogy clinic
spt ini disebut apa? because of some nodules on his arm and body
and sometimes feel pain with pressure. On
A. Early treatment failure examination was found erythematous
B. Late treatment failure nodules, smooth and shiny diffused infiltrate
C. Late paracitemia with ill define border, no flunctuation or
D. Early paracitemia erosion. There were pain on his elbow with
E. Late manifestation nerve enlargement. Diagnosis?
a. yaws
4. Orang datang ke klinik, demam tinggi dll b. leprosy (ada nerve enlargement bagian elbow
(malaria), manifestasi klinis pasien ini & lutut
terjadi pada saat nyamuk dlm stadium c. anthrax
apa? d. cellulitis (jawaban 2012. Yakin neh nggak
ngaco???)
A. Tropozoit e. carbuncle
B. Sporozoit
C. Merozoit 11. A laboratory examination of a leprosy patient
D. Mikrogametosit after 4 months MDT therapy revealed
E. Makrogametosit Bacterial index 6+ and morphological index
50%. What is your conclution for this result ?
5. Pasien mengalami demam tinggi pada
hari ke 4 dan didiagnosis malaria, a. Pausibacillar leprosy patient sensitive to
termasuk malaria golongan apa? Malaria treatment
Malariae b. Multibacillar leprosy patient sensitive to
treatment
c. Multibacillar leprosy patient resistent to arm and legs also fever. What is the most likely
treatment diagnosis ?
d. Pausibacillar leprosy patient resistent to
treatment a. Celulitis
e. Pausibacillar leprosy patient prone to nerve b. Furuncle
damage c. Erythema nodosum
d. Multibacillar leprosy
12. Yang harus di considerate untuk pasien e. Pausibacillar leprosy\
suspect leprosy? Lesi di tubuh yg painless &
numb 17 Residen mengeluh karena di ruangannya AC
terlalu dingin, suara berisik dan lampu terlalu
13. Pasien kena malaria, best drug of choice: silau. Termasuk hazard kategori apa? Physical
hazard
A. Doxycicline
B. Chloroquine 18 A 10-year-old boy who has moved to your
C. Mefloquine practice recently has sore throats, get high
D. ACT temperature. The child reports a runny nose, mild
cough, and abdominal pain. Findings on physical
14. G3P2A0 gejala anak chorioretinitis, cerebral examination include a temperature of 100.3°F
calcification, hepatomegaly, splenomegaly, (38°C), and vesicular lesions on the soft palate.
berat badan lahir rendah. Ibunya dalah suster There is no cervical adenopathy or rash. Of the
di RS siloam, tidak pernah kontak dengan following, the MOST likely diagnosis is
kucong. Diagnosis?
a. Adenovirus infection
a. congenital toxoplasmosis b. Coxsackie virus infection
b. congenital CMV c. Mononucleosis
c. congenital rubella d. Sinusitis
d. congenital herpes e. Streptococcal pharyngitis

15. a baby is born with a rash identical. A 2 years old boy brought to dermatovenereology
Questioning reveal that the mother had a febrile clinic because of itching all of the body. He
illnes during the second teimester of pregnancy. complains itching on palm, side of finger,
Examination reveals diffuse raised purple skin buttock, and abdomen. Clinical manifestations
lesions. There is no pallor, jaundice, or cyanosis. are papule, vesicle, pustule, erotion, and
The eyes are cataract, the heart has a 3/6 systolic excoriation especially at finger web, arm,
murmur and there is enlargement of both the abdomen, side of hand and feet, itchy especially
liver and spleen. There is no lymphadenopathy. at night.His neighbours have same problem.He
Of the following, the most likely diagnosis is? usually play together with his neighbours.What is
the diagnose this patients?
a. congenital toxoplasmosis
b. congenital rubella a. Pediculosis corporis
c. congenital CMV b. Scabies
d. congenital malaria c. Tinea corporis
e. congenital varicella d. Dermatitis atopik
e. Psoriasis
16 A baby is born with a rash identical. History
taking reveals that the mother had a febrile illness 20 A patient on MDT treatment complaining
during the second trimester of pregnancy. about red urine that he experinced on the first day
Examination reveals diffuse raised purple skin of treatment. What is the suggestion for this
lesions. There is no pallor, jaundice, or cyanosis. condition ?
The baby has spread over his body especially on
a. Stop Clofazimin, can cause pink a. Salmonella typhi
brownish discolouration b. Plasmodium falciparum
b. Continue the medication, its not a drug c. Vibrio cholera
adverse effect d. Hepatitis A
c. Stop the medication because of e. Escherichia coli
nefrotoxicity
d. Stop Dapsone can cause anemia 24. Best laboratory examination to detect
hemolytic S.typhii? Blood smear
e. Change the treatment to single drug
therapy 25. Best treatment for scabies? Gamexane

21. A 27-year-old female has just returned 26. Tukang kebon dtg ke klinik dgn keluhan
from a trip to Southeast Asia. In the past demam tinggi sampe 39,5 derajat,
24 hours, she has developed shaking, profuse sweating, mual muntah, best lab
chills, and a temperature 40o C. A blood examination? (suspect malaria)
smear reveals Plasmodium vivax. Which
of the following agents should be used to A Blood culture
eradicate the extraerythrocytic phase of B Stool
the organism? C Saliva
D
a. Primaquine E
b. Pyrimethamine
c. Quinacrine 27 Transmisi penyakit yg mungkin utk kasus
d. Chloroquine diatas? Direct skin scale contact (digigit nyamuk)
e. Chologuanide
Anak demam 2 hari, dokternya suspek DBD.
22. Ibu hamil mau bepergian ke Papua, obat mau cek apa?
prophylaxis yg cocok buat dia? A. NS1
B. Hematocrite
Mefloquine C. IgM
(nggak boleh dikasih doxi & kalo dikasih
chloroquin percuma krn mungkin
resisten) 28 Workplace health hazard can cause
reaction. Asthma, and dermatitis caysed
by whicch reaction?
23. A 26-year-old nulliparous woman, in her
seventh month of pregnancy complains a. immediate reaction
of a 7 days history of fever especially at b. acute
night. As she recall, 2 weeks ago she ate c. gradual
gado-gado in coastal area when she out d. delayed
for duty for several days. She also had e. no reaction
diarrhea and abdominal discomfort. On
physical examination reveals blood 30 cowo aktif sexual partner tapi pake pelindung,
pressure 110/70 mmHg, heart rate 60 terus mau screening. Screening pake apa?
×/min, RR 20x/min, temp. 39.1°C.
Tongue is coated and tremor. Fetal USG a. HIV-RNA
shows normal. b. CD4 count
c. Rapid test
What is the most likely etiologic factor d. western blot
above? e. HIV-ELISA
involves the truncal muscles. Possible
31 Elephantiasis is caused by the obstruction of : diagnsosis?
a. The arteries by microfilariae
b. The arteries by adult worm a. Guillain-Barré syndrome (ascending
c. The lymphatics by microfilariae paralysis)
d. The chronic lymphatics by adult worm b. Poliomyelitis
e. Fibrosis of the lymphatic c. Encephalitis
d. Meningitis
32 A 40-year-old man came to the clinic with e. Multiple Sclerosis
complaints of having fever more than one-week,
nausea, and fatigue. He came back from Timika, 36 A 53-year-old male farmer developed low-
Papua. On blood smear examination, there are grade fever, abdominal pain, and diarrhea. He
late trophozoites forms, schizonts with 12-24 has a skin lesion of black eschar surrounded by
merozoites, and the infected red cell is vesicle and edema near his mouth. A week
enlarged.Diagnosis? P vivax before, his notice many cattle in his farm were
sick, but he still eats a half-done beef steak
33 A 10-month-old healthy male infant from his cattle.
travelling with his family to Africa was
exposed 4 days ago to his 4-year-old native What is the following microorganism is most
African cousin who was ill at the time with likely involved in this case?
fever, cough, coryza and conjunctivitis. The a. Bacteroides fragillis
baby was at risk to get infected by: b. Leptospira interrogans
a. Corynebacterium diphtheriae. c. Bacillus anthracis (bakteri gram positif)
b. Haemophilus influenzae d. Pasteurella pestis
c. Bordetella pertussis e. Borrelia burgdorferi
d. Morbilivirus measles
e. Coxsackievirus 37 cowo, MSM, datang ke poliklinik mau
evaluasi treatment HIV nya. 1 minggu lalu, HIV
34 A 65-year-old-woman suddenly had + dan CD4 <350. Next step management?
paralysis of her left leg. A few days ago, she
had headache, fever, sore throat, and nausea. a. start arv
She lives with her son; daughter in law and a b. cek tb
young grandchild who’s just received a routine c. kasi prophylaxis
oral vaccination. She is taking d. rapid test
immunosuppressant for her kidney’s transplant. e. do nothing
Her vital signs and cranial nerve examination
are normal. A head CT scan and lumbar MRI 38 52- year old man is brought to the
are also normal. emergency department in an unconscious
condition. On examination, he was death form
What is the most likely transmission of her more than one hour. On autopsy of the brain,
infection? there is cyst‘s fluid containing protoscoleces
a. Droplet with hooklets and many daughter cysts were
b. Air borne found. What is the most likely diagnosis of this
c. Fecal-oral (suspect polio) disease?
d. Close contact
e. Animal bite a. Hydatidosis
b. Cysticercosis
35 A 5 year old boy was brought to emergency c. Cerebral malaria
department with complaining of paresthesia of d. Hstoplasmosis
the fingers and lower extremity muscle weakness e. Toxoplasmosis
since two days ago. The weakness progress
39 A 27 year old women suffering fever for 2 d. B. Parapertussis
day. She brought to emergency department e. C. diphtheriae
unconsciously and she had general seizure 2 hour
ago. On physical examination she is comatous, 42 What is the laboratory test to confirm the
GCS (Glasgow Coma Scale) : 3; BP 90/60 diagnosis in this patient?
mmHg; t 41 °C. Lab. finding: Hb 10, 3 gr%;
WBC 5800/mm3; platelet 39.000/mm3; there is a. Blood culture
schizon form on blood smear. Why the b. Throat swab culture
pathogeniv agent of this disease becoming the c. Chest X-ray
most dangerous species? d. Measurement of IgG antibody serum
e. Urine culture
a. It make long term relaps
b. Intermittent fever 43 A 70 years old women, come to
c. Gametocyte formation take place in dermatovenereology clinic because she
visceral organ complains pain, edema on her left eyelid and
d. Can make capillary obstruction in Central there is some vesicle in her left forehead. She
Nervous System by parasite contain complain pain on her forehead 2 days ago, but
erythrocyte vesicle come out 1 day ago.She sometimes fells
e. There are many drug resistance against P. headache and fever. Clinical manifestation are
falciparum grouped vesicle, eritema base at her left
face.What is the diagnose of this patient?
40 A female tourist developed
gastroenteritis while visiting small a. Varicella
town in Indonesia and tried some b. Herpes zoster
Indonesian traditional food. The onset c. Herpes simpleks
of the disease is abrupt with abdominal d. Dermatitis venenata
cramps and watery diarrhea. She had e. Dermatitis iritan
no fever or nausea or vomiting. The
symptoms have resolved within 24 44 A 19-year-old woman comes to
hour and no subsequent recurrences. Emergency Department with signs and
They report the disease to district symptoms of poisoning of an agent include
public health office. The investigation nausea, vomiting, abdominal cramps,
found that one of the food products diarrhea, excessive salivation, headache,
eaten by this tourist was contaminated giddiness, rhinorrhea, tightness in chest, pin-
by suspected pathogens. What is the point pupils parasimpatetik activity, mental
suspected pathogen may cause the confusion, and muscle twitching. Which of
disease above? the following is the most likely agent caused
a. Salmonella typhi those symptoms?
b. Shigella dysenteriae
c. Enterohemorragic E. coli a. Acetaminophen
d. Staphylococcus aureus b. Barbiturate
e. Enterotoxigenic E.coli
c. Carbamate
41 A 3-year-old boy was brought to emergency d. Cyanide
unit in Tangerang Hospital with stridor, dyspnea, e. Opiate
and “croupy” cough, What is the most likely
pathogenic agent that causes the patient’s 45 Orang demam tinggi, nyeri belakang mata,
disease? demam, Hb Ht naek, berdasarkan WHO,
a. Bordetella pertussis diagnosisnya apa?
b. Haemophilus influenzae
c. Coxsackievirus A. Dengue fever
B. DHF (hemokonsentrasi & tanda disentri ; stadium 3 sirosis hati &
perdarahan) splenomegaly)
C. Severe Dengue
a. creeping eruption
46 49 yo man come to dermatologist because of b. billiary cirrhosis (gw jawab ini)
creeping in left elbow on day 3 trip. Eruption c. cholecystitis (jawaban 2012, yakin nih?)
6cm, linear, erythematobolbulus. Most possible d. pruruitus ani
causees? e. visceral larva migran

a. dermacentor andersoni 52 a 23yo man presents with extreme swelling of


b. lactrodectus mancans his legs and scrotum. The skin associated with the
c. loxoxsceles laeta swollen areas is thick and scally. The patient
d. lytta sesuatu admits to an episode of fever associated with
e. paederus peregrinus (tomcat) enlargement of inguinal lymph nodes some times
ago, but did not think much of it. Which of the
47 A 44-year old man on a business trip to South following is the best method to make diagnosis?
Kalimantan from Jakarta presented to the
emergency department with symptoms of a. examine the stool for presence of eggs
diarrhea and abdominal pain. On physical b. examine the stool for the presence of
examination was found pitting edema of the larvae
abdominal wall and of the legs. A stool specimen c. examine the blood smear for the presence
was sent to the laboratory for culture and for ova of adults worm
and parasites. Blood was drawn for a CBC. The d. determine the titer of IgE antibody
culture was negative for enteric pathogens. The against the parasites
patient was found to have eosinophilia. The wet e. examine the blood smear for the presence
mounts from the concentrated stool specimens of microfilaria
revealed large, ellipsoidal, operculated parasite
eggs with thin, transparent shells (140 x 85 µm). 53 Treatment yg cocok utk case diatas (suspect
filariasis)? DEC (Diethylcarbamazine Citrate)
Of the following, which parasite can be the =>wkt kt bljr sempet gw sebut ini
cause of this disorder :
a. Schistosoma japonicum 54 A 40 year old man came to hospital with four
b. Fasciolopsis buski times generalized tonic-clonic seizure since two
c. Trichinella spiralis week ago. There was no history of seizure and
d. Echinococcus granulosus head trauma. A one year ago, he had business in
e. Clonorchis sinensis Papua for six months. During his stay in there, he
48 hematuria dan oval shaped and ada di terminal often consumed under-cooked pork. Brain MRI
spine? showed a small cystic with invaginated scolex
schistosoma hematobium inside. What of this following is the most likely
inside cyst?
49 40yo berenang di danau lindu, gatal2, besok
muncul papul, most likely pathogen? a. Taenia eggs
schistosoma japonicum b. Cycticercosis cellulose
c. Cysticercosis bovis
50 Bentuk infeksi yg nyerang pasien tsb (soal d. Taenia solium worm
49): larva filariform e. Taenia saginata worm

51 Manifestasi klinisnya: (dari buku UI: 55 Possible etiology case di atas? Undercooked
stadium 1 urtikaria, kalo intoksikasi pork
disertai demam, hepatomegaly &
eosinophilia tinggi ; stadium 2 sindrom 56. a 39 yo woman whose husband has a pig at
home kept for house purposes come to the family
doctor with symptoms of periorbital edema, 61 56 year old sheep farmer presents with
weakness and muscle pain. What is the most abdominal pain. Liver is enlarged with
probable parasite causative in this case? palpable mass at right upper quadrant.
CT scan reveals a large encapsulated
a. schistosoma japonicum lesion consistent with HYDATID
b. f. buski CYST. What organism may have cased
c. trichinella spiralis this? Echinococcus granulosus
d. echinococcus granulosus
e. clonorchis sinesis 62 manifestasi apa yang paling berbahaya
ketika surgucal removal case di
57 Etiologic cause case di atas? atas?bayangin aj, ni soal di angkatan lu
a. eating larvae in contaminated food sempet keluar tp ga ada yg nyatet
b. eating eggs in contaminated food atasnya. Anjeng bener emg…!!!!!!!!!!!!!
c. eating cyst in contaminated food
d. undercooked pork a. seizure
e. undercooked beef b. hemorrhage
c. peritonitis
58 A patient complains of having nail size d. anaphylactic shock
excrement from his anus. On the stool e. cardiac arrest
examination, parasite eggs contains a
hexacanth embryo with six hooklets
surrounded by radially striated spherical shell, 63 The patient was 3 y.o girl who was
30 to 40 u in diameter were found, the most seen by her pediatrician for a routine
likely diagnosis of this patient is : physical exam. Her mother was
concerned about her daughter’s poor
a. Saginata taeniasis (di feses biasa appetite. Physical exam revealed that
ditemuin proglotid) the child was small for her age and had
b. Solium taeniasis a slightly enlarged liver. Blood was
c. Cysticercosis collected for a routine CBC, since she
d. Oxyuriasis had previously been slightly anemic.
e. Ascariasis Hb normal, eosinophilia, no travel
history, melihara anak anjing di rumah.
59 a 14 yo boy fell from bicycle, Possible cause? Toxocara canis
last immunization at 18month,
(4th doses) 4-6 year 64 Manifestasi klinis case di atas? Visceral
immunization wasn’t done. larva migrans
What terapy should be given?

a. tetanus Ig 65 A 40-year-old man came to the clinic


b. human iv Ig with complaints of having fever more
c. tetanus toxoid than one-week, nausea, and fatigue. He
d. anti tetanic serum came back from Timika, Papua. On
blood smear examination, there are late
trophozoites forms, schizonts with 12-
60 Cara ngambil specimen darah yg baik? 24 merozoites, and the infected red cell
Gw lupa pilihannya, pokoknya di is enlarged.Diagnosis? P. vivax
handout dia blg: diambil saat yg tepat
(cthnya pas suhu naek), peralatan yg 66. Cewe, flank pain, pas urinalysis hasilnya
baik, ambil pas pasien blm minum positive nitrate dan many leucoytes, next
antibiotic step>
a. urine culture d.
b. PCT oral e. leptospirosis
c. USG
d. .
e. IV ceftriaxone 70 A 35-year old man working on pig farm is
brought by his employer to the emergency
67. 9 years old susah nafas, noisy breathing department of Siloam hospital because of seizure
sejak 3 hari yang lalum juga ada fever cough and bilateral lower extremity weakness. CT scan
myalgia headache hoarseness. Refuse to eat
karena sakit tenggorokan, ada pembesaran of the head reveals several calcified regions.
KGB leher, white membrane di tonsil. Laboratory examinations show WBC count of
Additional exam? 10.800/mm3, with eosinophils. Brain tissue
f. Neck xray to reveal laryngeal biopsy reveals scolex with hooklets. Which one is
edema
g. CBC – liat infeksi bakteri the treatment of choice for this case?
h. Throab swab – untuk gram
staining danswab kultur a. Sulfadiazine-pirimetamin
i. . b. Artemisinin
j. refer THT
c. Albendazol
68 A boy brought by his mother to the d. Metronidazol
emergency department of Siloam hospital e. Itrakonazol
because of vomit & dyspnoea. According to his
mother, her son was playing in the backyard. On 71 A 35-year-old man as a farm worker who
physical examination, the boy was found to have was working with pesticides is brought to the
high fever (39 ºC) and multiple stinging on the emergency room with headache, vomiting,
back. The most likely species of insect bites is : salivation, diarrhea, muscle fasciculation,
difficulty walking, and difficulty speaking. His
a. Hymenoptera sp clothing has been removed, he has been
b. Dermacentos andersoni washed, and he has been given activated
c. Lactodectus mactan charcoal. What is the most effective remaining
d. Loxoxceles laeta treatment for this case of pesticide poisoning?
e. Lytta vesicatoria
a. Epinephrine
b. Antacid
c. Spironolactone
69 21yo man come to emergency department d. Atropine
due to high fever, headache, nausea and e. Hidrochlorothiazide
vomiting and constant sleepiness for 5 days. At
72. Occcult filariasis is Lymphatic filariasis
admission, he look lethargic and dehydrated. based upon
At PF, he look jaundice, conjuctival injection a. infection
and … there are liver dysfunction and b. allergy
increased creatinine. The relative stated that c. infestation
d. colonization
the family live in village, tempoarte climate. e. genetic
Previous 10 days, he worked on irrigation
channel and isinya air sungai. What is your
diagnosis? 73. frustated patient diagnosed with leprosy.
He is afraid that he is cursed abd going to die.
a. treponemasis As a medical doctor, what will you say to the
b. shigelosis patient?
c. salmonelasis
b. Lepidopterism
a. rest all day and don’t do his job c. Tick paralysis
b. cannot be cured d. Arachnidism
c. treatment in leprocy house e. Delusional parasitosis
d. look out for wound
e. highly contagious and contact his family
79 Orang HIV, ada riwayat TB dan Hepatitis
B. kombinasi obat yg dipake? Gw lupa, tp gw
74 a patient come with an itchy recured jawabnya yg Emicitabine, Tenovovir,
fungal infection on feet, doctor already give Efavirenz. Pelajarin sendiri aja ye?
him some medication for fungal infection,
what can be advised to make him healthy? 80 a 45 yo women, ke rs karena diare 3 hari,
tiap hari 5-6x, sebelumnya dirawat di rs karena
a. don’t eat seafood pneumonia. Pernah ada history nasopharunx
b. always use sandals CA, faktor yg mempengaruhi diare?
c. keep the skin dry and clean
d, don’t use shoes all day long a. agent not usually cause disease in healthy
e. don’t scratch, it will spread the infection person
b. evolution of pathogen shg bisa ke new host
75 Komplikasi demam thypoid paling c. development of drug resistant of pathogen
sering dimana? Ileum distal (pilihannya d. non-adherence pasien dlm minum penumonia
wkt itu ada colon gt2) drugs
e. pake counterfeit drugs
76 Cewe kena tusuk jarum suntik orang yg kena
HIV, langkah selanjutnya? 81. Anak sekolah diperiksa ada gatel2 dll (gejala
scabies), trus temen2nya di kelas jg ngalami
A. Rapid test keluhan yg sama. Langkah yg seharusnya
B. Post-exposure HIV prophylaxis (gw dilakukan?
jawab ini)
C. Start antiviral A Preventif
D. Do nothing B Kuratif
C Rehabilitatif
77 cewe dtg dgn keluhan bengkak di D Promotif
tungkai bawah, thick blood smear ada E Environmental modification (gw nembak
microfilaria ciri deeply stain giemsa, inti jawab ini)
sampe ke ekor dan teratur, perbandinga
kepala ekor 2:1, parasit apa?
TROPMED 2012
a. wuchereria bancrofti
b. brugia malayi (thio) Kerja di poultry dan ayamnya pada mati.
c. brugia timori 35 tahun, ada gejala influenza, apa virusnya?
d. onchocerca volvulus
a. orthomyxoviridae
e. loa loa

78 One member of the NGO (Non- (dion) anak luka trus uda di ksi vaksin tetauns,
Governmental Organization) team from tapi hanya sekali. Management skrg apa?
Indonesia has been reported dead in Mexico
because of a spider’s bite. The possible cause of Jawaban : Tig
this disorder is ;

a. Erucism
(guntur) 24 thn skoleks ditemukan di CT scan c. peritonitis
atau MRI.
d. anaphylactic shock
a. taenia solium
b. b. toxo e. cardiac arrest
c. c. echinococcus

2. salmonella typhii, lab diagnosis? 14. manifestation of infection above:

a.stool culture a. creeping eruption

b. blood culture b. biliary cirrhosis

c. swab c. cysticercosis

d? d. pruritus ani

e? e. visceral larva migrant

3. lab exam buat salmonella? 15. a 39 yo woman whose husband has a pig at
home kept for house purposes come to the
a. IgM salmonella family doctor with symptoms of periorbital
edema, weakness and muscle pain. What is the
b. anti HCV most probable parasite causative in this case?

a. schistosoma japonicum

7. which subtype are known to cause HPAI? b. f. buski

a. H1 and H3 c. trichinella spiralis

b. H2 and H4 d. echinococcus granulosus

c. H5 and H7 e. clonorchis sinesis

d. H6 dan H8

e. H9 and H11 16. atasnya soal trichinella spiralis?

a. eating larvae in contaminated food

8. hep B positive, HIV positive, tb positive, b. eating eggs in contaminated food


treatment untuk HIV? TEE
c. eating cyst in contaminated food

d. undercooked pork
12. dari kasus 11, manifestasi apa yang paling
berbahaya ketika surgucal removal? e. undercooked beef

a. seizure

b. hemorrhage 17. ada org datag dengan demam lebih dari 1


minggu. Dari blood smear ada parasit bentuk
cincin dgn pembesaran sel darah merah. Kena c. brugia timori
apa?
d. onchocerca volvulus
a. plasmodium vivax
e. loa loa
b. p. malariae

c. p. falciparum
26. frustated patient diagnosed with leprosy. He
d. p. ovale is afraid that he is cursed abd going to die. As a
medical doctor, what will you say to the patient?
e. p. knowlesi
a. rest all day and don’t do his job

b. cannot be cured
18. demam tiap hari ke-4
c. treatment in leprocy house
jaw : plasmodium malariae
d. look out for wound

e. highly contagious and contact his family


21. treatment no 20

a. metronidszole
28. a patient come with an itchy recured fungal
b. dietilcarbamasin infection on feet, doctor already give him some
medication for fungal infection, what can be
c. mebendazole advised to make him healthy?

a. don’t eat seafood


23. 49 yo man come to dermatologist because of b. always use sandals
creeping in left elbow on day 3 trip. Eruption
6cm, linear, erythematobolbulus. Most possible c. keep the skin dry and clean
causees?
d, don’t use shoes all day long
a. dermacentor andersoni
e. don’t scratch, it will spread the infection
b. lactrodectus mancans

c. loxoxsceles laeta
29. a 45yo man was seen in the dermaotlogy
d. lytta sesuatu clinic because of some nodules on his arm and
bpdy and sometimes feel pain with pressure. On
e. paederus peregrinus examination was found erythematous nodules,
smooth and shiny diffused infiltrate with ill
define border, no flunctuation or erosion. There
25. cewe dtg dgn keluhan bengkak di tungkai were pain on his elbow with nerve enlargement.
bawah, thick blood smear ada microfilaria ciri Diagnosis?
deeply stain giemsa, inti sampe ke ekor, parasit
apa? a. yaws

a. wuchereria bancrofti b. leprosy

b. brugia malayi c. anthrax


d. celullitis d. paracitemua dri repeated cycle schizon from
liver to erytocyte
e. carbuncle

36. treatment malaria


31. a 21 yo man was seen in dermatology clinic
because of sore and redness on his previous skin a.
lesion. He was on MDT treatment since 1.5 year
ago. On skin examination found some painful b. chloroquinee HCl intramuscular
erythematous nodule spread over his body
especially on arm and legs also fever. What is c. artemeter + lumefanfrine
the most likely diagnosis? d. primaquiine
a. celullitis

b. furuncle 37. a 45 yo women, ke rs karena diare 3 hari,


c. erythema nodosum tiap hari 5-6x, sebelumnya dirawat di rs karena
pneumonia. Pernah ada history nasopharunx CA,
d. multibacillar leprosy faktor yg mempengaruhi diare?

e. pausibacillar leprosy a. agent not usually cause disease in healthy


person

b. evolution of pathogen shg bisa ke new host


33. patient on MDT treatment complaining
about red urine that he experience on first dat of c. development of drug resistant of pathogen
treatment. Suggestion?
d. non-adherence pasien dlm minum penumonia
a. stop clofazimin, can cause pink brownies drugs
discoloration
e. pake counterfeit drugs
b. continue medication, it is not drug adverse
effect

c. stop the medication because of nephrotoxicity 39. a 14 yo boy fell from bicycle, last
immunization at 18month, (4th doses) 4-6 year
d. stop dapsone can cause anemia hemolytic immunization wasn’t done. What terapy should
be given?
e. change the treatment to single drug therapy
a. tetanus Ig

b. human iv Ig
35. upper quadrant pain dan fever. Liver
enlargement and tenderness. Baru pulang dari c. tetanus toxoid
thailand. Chill and sweat waktu fever. Terlihat
3.5% paracitemia. Explain mna yang benar? d. anti tetanic serum

a. paracutemia … from tissue injury

b. paracitemia release of merozoit 41. 6yo girl was brought to clinic, cough,
vomiting. PE found: crackles in both lungs.
c. paracitemia due to sporozoit initiate in human Treatment?
infection
50. a 19yo woman come to emergency
deoarment with sugn and sympotms of
42. 24yo woman datang ke dokter pribadi mau poisoning of an agent include nausea, vomiting,
ke papua selama 1 minggu. Ibunya uda menikah, abdominal cramp, diarrhea, excessive salivation,
2 bulan gk haid, diksi prophylaxis apa? headache, giddiness, rhinorrhea, tighness in
chesr, pin-point pupil, mental confusion and
a. sulfa- piremitamin 2 weeks before muscle twitching. Which of the following is the
b. an… most likely agent caused those symptoms?

c. doxycycline 2 days before a. acetaminophen

d. mefloquine 2 days before b. barbiturate

e. primaquine 2 weeks before c. carbamate

d. cyanide

44. management soal no 43: e. opiate

a. hospitalisation with iv

b. outpatient meal soup 51. 40yo berenang di danau lindu, gatal2, besok
muncul papul, most likely pathogen?
c.
c. schistosoma japonicum
d. jambu biji

e. domperidone
53. soal no 51 ditanta bisa nyebapin apa?

a. creeping eruption
48. G3P2A0 gejala anak chorioretinitis, cerebral
calcification, hepatomegaly, splenomegaly, berat b. billiary cirrhosis
badan lahir rendah. Ibunya dalah suster di RS c. cholecystitis
siloam, tidak pernah kontak dengan kucong.
Diagnosis? d. pruruitus ani
a. congenital toxoplasmosis e. visceral larva migran
b. congenital CMV

c. congenital rubella 55. makan gado2, trus ever, lidah coated, termor.
Causative agent menyebabkan perforasi dimana?
d. congenital herpes
a. ileum

49. hematuria dan oval shaped and ada di


terminal spine? 57. a 23yo man presents with extreme swelling
of his legs and scrotum. The skin associated with
Jaw : schistosoma the swollen areas is thick and scally. The patient
hematobium admits to an episode of fever associated with
enlargement of inguinal lymph nodes some
times ago, but did not think much of it.
Which of the following is the best method to 64. 8 day old infant, any loud noise appears to
make diagnosis? cause him pain, ada muscle tightening and back
arching, causing the head to nearly touch his
a. examine the stool for presence of eggs feet. PF reveals only drued packing on his
b. examine the stool for the presence of umbilical cord as is the local custom. He appear
larvae normal until stimulated by loud noise ot touch
c. examine the blood smear for the and then began to cry, stiffens and arches his
presence of adults worm back. The stiffness continues until he calm
d. determine the titer of IgE antibody down. Most likely diagnosis:
against the parasites
e. examine the blood smear for the a. bacterial meningitis
presence of microfilaria
b. botulism

59. cowo, MSM, datang ke poliklinik mau c. generalized seizure


evaluasi treatment HIV nya. 1 minggu lalu, HIV
+ dan CD4 <350. Next step management? d. TETANUS

a. start arv e. viral encephalitis

b. cek tb

c. kasi provilaksis cotrim 67. ibu bawa anak umur 18 bulan ke ER dgn
keluhan runny nose, conjuctivitis, diarrhea. PF:
d. jgn kasi arv sampao cd4 <200 fever 40C 3 hari, rash 2-3mm mulai dari muka.
Most likely causes:
e. kasi prophylaxis clatromycin
jawban: morbili

63. 2yo brought to dermatovenereology clinic


because itching all over the body. He 69. a baby is born with a rash identical.
complained of itching on palm, side of finger, Questioning reveal that the mother had a febrile
buttock and abdomen. Clinical manifestations illnes during the second teimester of pregnancy.
are papule, vescicle, pustule, erotion and Examination reveals diffuse raised purple skin
excoriation, especially at finger web, arm, lesions. There is no pallor, jaundice, or cyanosis.
abdomen, side of hand and feet. Itchy especially The eyes are cataract, the heart has a 3/6 systolic
at night. His neighbour have the same problem. murmur and there is enlargement of both the
He usually play together with his neighbour. liver and spleen. There is no lymphadenopathy.
What is the diagnosis of this patient? Of the following, the most likely diagnosis is?

a. pediculosis corporis a. congenital toxoplasmosis

b. scabies b. congenital rubella

c. tinea corporis c. congenital CMV

d. dermatitis atopik d. congenital malaria

e. psoriasis e. congenital varicella


71. ke india 6 hari lalu, foul smelling and b. CD4 count
slimmy diarea
c.
a. giardia lamblia
d. western blot
b. vibrio cholera
e. HIV-ELISA
c. shigella

77. 67 yo man with long lasting diabetes and


72. cewe 55 thn, mau ke makkah, dia uda 10 abscess in pedis dextra. PF BP 120/80, HR
tahun gak kesna, pemeriksaan fisik normal, 120bpm, RR 28, tempt 37,8C. Hb 8, WBC
perlu vaksin apa? 21,000, platelet 109,000. Next step evaluation
and management?
a. salmonella typhii
a. administer normal saline bolus
b. tetanus
b. obtain culture
c. hepatitis a
c. transfusion red pack blood cells
d. hepatitis b
d. platelet transfusion
e. meningiococcal
e. bone marrow biopsy

73. 21yo man come to emergency department


due to high fever, headache, nausea and 78. boy 12yo dtang ke klinik demam 4 hari yg
vomiting and constant sleepiness for 5 days. At lalu, semam, mual, skait kepala, fatique,
admission, he look lethargic and dehydrated. At tourniquet +, apa arti hasilnya?
PF, he look jaundice, conjuctival injection and
… there are liver dysfunction and increased a. increased capillary liability
creatinine. The relative stated that the family
live in village, tempoarte climate. Previous 10 b. devreased platelet count
days, he worked on irrigation channel and isinya c. increase hematocrit
air sungai. What is your diagnosis?
d. decrease platelet function
a. treponemasis
e. DHF
b. shigelosis

c. salmonelasis
79. anak demam 2 hari, dokter suspek DBD,
d. cek?
e. leptospirosis a. NS1

b. hematocrite
75. cowo aktif sexual partner tapi pake c.IgM
pelindung, terus mau screening. Screening pake
apa?

a. HIV-RNA
80. 38yo woman fever 10 days, malam makin
parah, ada fatique, dizzines, stomach discomfort,
constipation, loss of apetite. Suhu 38C BP
100/70 pulse 70x. mild abdominal pain, tremor
and coated tongue. Main specific standart lab
exam for diagnosis?

a. PCR

b. Widal

C. organism culture isolation

81. workplace health hazard can cause reaction.


Asthma, and dermatitis caysed by whicch
reaction?

a. immediate reaction

b. acute

c. gradual

d. delayed

e. no reaction

85. nenek dtg kidney transplant tinggal dgn anak


n cucu. Anak bayi baru vaksin. Nenek merasa
kaki lemah sebelah. Vaksin apa bisa prevensi
penyakit ini?

a. HiB

b. orosabin

c. meningiococcal

d. varicella

e. hep B
MCQ TROPICAL MEDICINE 2010 a. Stool
b. Blood
1. An apparently run-down but alert 34-year- c. Saliva
old woman comes to your office after 6 d. Lung biopsy
months spent as a teacher in a rural West e. Nasopharyngeal swab
Sumba. Her chief complaints are frequent
headaches, occasionally nausea and 4. A 16-year-old boy presents with an
vomiting, and periodic fever. To rule out annoying cough, which he has had for about
your differential diagnosis, a smear of 3 weeks. The illness started with a runny
finger-stick blood was done. Which of the nose. On examination he is completely
following choices would fit your diagnosis normal, but he exhibits several episodes of
based on your microscopic examination of severe coughing. He has difficulty to take
the blood smear? deep breath and usually ends up with
a. Schizonts in red cells with 8-12 progeny vomiting. Of the following, the MOST
b. Rounded gametocytes present likely diagnosis is
c. Enlarged, somewhat misshapen in red a. Tonsilitis
cells b. Bronchiolitis
d. Large ovoid parasites in some of the red c. Pertusis
cells d. Laryngitis
e. Band-shaped tropozoites in infected red e. Diphteria
cells 5. A young man was admitted to hospital after
increasing left arm pain and paresthesia.
2. A 20-year-old woman came to the Several days ago, he was bitten by a dog.
emergency room with high fever from 5 His symptoms increased and were
days ago. She also has headache and pain in accompanied by hand spasm and sweating
retroorbita, abdominal pain and bloody on the right side of the face and trunk. This
stool. On physical examination, she was in patient was admitted to the hospital the day
an agitated condition. Her BP 90/60 mmHg, after developing dysphagia, hydrophobia,
heart rate 110x/ minute, RR 20x/ minute and hypersalivation, and disorientation.
there were purpura in her arm, hand and leg. Which of the following pathologic feature is
Which of the following examination is the most likely found in dog’s brain?
most appropriate to establish the diagnosis? a. Giant cell
a. Prick test b. Limfosit plasma biru
b. Dengue IgM-IgG c. Negri bodies
c. Complete blood test d. Inclusion bodies
d. Widal test e. Clue cell
e. Stool analysis
Question 6-7 are linked to the following case:
3. A 32-year-old female developed a “flu-like” A 32-year-old worker from a farm comes with a
syndrome with high fever up to 400C, painless dermal papule on his right hand which
anorexia, headache, and myalgia. Four days started to develop since five days ago. The
later, she become confused and agitated with worker says as the lesion getting bigger, and the
difficult to breath. Chest x-ray showed skin becomes black . He has tender axillaries
parenchymal infiltrate. One of her kids has lymph node enlargement.  anthrax
also these symptoms, and has already passed
away. In order to provide laboratory 6. What antibiotic should the doctor give to
conformation of the pathogenic agent, a treat the worker?
tissue culture was ordered. a. Tetracycline
Which of the following would be the best b. Aminoglycoside
specimen for isolating the pathogenic agent c. Ciprofloxacin
responsible for this infection? d. Neomycin
e. Crystalline penicillin G mouth. A week before, his notice many
cattle in his farm were sick, but he still eats
7. Which of the following virulence factors is a half-done beef steak from his cattle.
most likely to be involved in the What is the following microorganism is
pathogenesis of illness? most likely involved in this case?
a. Exotoxin a. Bacteroides fragillis
b. Endotoxin b. Leptospira interrogans
c. α-hemolysin c. Bacillus anthracis
d. Lipopolysaccharide d. Pasteurella pestis
e. Antiphagotic factors e. Borrelia burgdorferi

8. A 45-year-old man was seen in the 11. A woman, recently returned from Lampung,
dermatology clinic because of some nodules complains of having paroxysmal attacks of
on his arm and body and sometimes feel chills, fever, and sweating; these attacks last
pain with pressure. On examinations we a day or two at a time and recur every 36 to
found erythematous nodules, smooth and 48 h. Which of the following is the most
shiny, diffuse infiltrate with ill define appropriate time to take blood specimen to
border, no fluctuation or erosion. There were establish the diagnosis?  malaria
pain on his elbow with nerve enlargement. a. Anytime
What laboratory test would you do to b. At 10.00 – 12.00 AM
confirm the clinical diagnosis?  leprae c. At 10.00 – 12.00 PM
a. TPHA and VDRL d. 30 minutes after fever
b. Gram preparation e. 120 minutes after fever
c. KOH preparation
12. A very depressed woman took 20 tablets of
d. Skin scrapping a hypnotic drug belonging to her husband
e. Skin slit smear about less than 30 minutes ago. As she was
being taken to the hospital, she developed
hypotension, tachycardia, and tachypneu.
She’s also unconscious. Which of the
following is the most appropriate
management for this patient?
a. Give apomorphine intramuscular 
induce emesis
9. A 40-year-old patient comes with the complaints b. Doing gastric lavage
of uncomfortable stomach & flatulence. From c. Infuse with sodium bicarbonate
history, the patient was known live in the valley d. Give activated charcoal via NGT
of Napu, Sulawesi and he had a fever frequently. (nasogastric tube)
On physical examination, his liver was found e. Doing a hemodialysis
enlarged. Which one of the following test can be
used to confirm the diagnosis?  typhoid
a. Blood test
b. Liver biopsy
c. Rectal biopsy
d. Stool examination
e. CT scan

10. A 53-year-old male farmer developed low-


grade fever, abdominal pain, and diarrhea. 13. An outbreak of pulmonary tuberculosis (TB)
He has a skin lesion of black eschar in a dormitory, and this outbreak were
surrounded by vesicle and edema near his presumed to be caused by the same source
of infection, based on the results of constipation. He did not receive any prior
restriction fragment length polymorphism vaccinations. His vital signs revealed
(RFLP) analysis and other findings. Which temperature 390C, BP 130/90, pulse 72x/min.
of the following health intervention is the His physical examination revealed mild
most appropriate for the case above? hepatospleenomegaly and faint erythematous
a. Rehabilitation macules.
b. Prevention
c. Curative 16. Which of the following is most likely to
d. Medication – ebola
havehemmorhagic fever
caused this man’s illness?
e. Environmental Modification a. Vibrio cholerae
b. Shigella dysentriae
14. There is an RNA virus that causes deadly c. Salmonella typhi
outbreaks of hemorrhagic disease in Africa. d. Entamoeba histolitica
The patents have developed fever, headache, e. Giardia lambia
and muscle pain followed by abdominal
pain, diarrhea, and rash, with both internal
and external bleeding. In each outbreak,
hospital staff became infected. This virus is
highly virulent. Which of the following is 17. Which of the following specimen is most
the most likely transmission for this disease? appropriate for case above?
 ebola a. Feces
a. Contact with blood or body fluid b. Blood
b. Transmitted by mosquitoes bite c. Urine
c. Transmitted to human from rodent d. Throat swab
excreta e. Biopsy
d. Aerosol transmission
e. Direct contact with cattle product Question 18-19 are linked to the following case:
A 44-year old man on a business trip to South
15. A 31-year-old man presented with Kalimantan from Jakarta presented to the
complaints of high fever, cough, and emergency department with symptoms of
shortness of breath. Two hours before diarrhea and abdominal pain. On physical
admission, he experienced right-sided chest examination was found pitting edema of the
pain when took a deep breath or coughed. abdominal wall and of the legs. A stool
Chest film showed diffused bilateral specimen was sent to the laboratory for culture
interstitial pulmonary infiltrate. Arterial and for ova and parasites. Blood was drawn for a
blood gases showed a PO2 of 60 mmHg with CBC. The culture was negative for enteric
91% hemoglobin saturation. The pathogens. The patient was found to have
hematologic, serum and liver tests were eosinophilia. The wet mounts from the
normal. He lives near the poultry, and the concentrated stool specimens revealed large,
pathogenic agent is a new virus. Which of ellipsoidal, operculated parasite eggs with thin,
the following is the prevention for this transparent shells (140 x 85 µm).
disease?
a. Vaccine 18. Of the following, which parasite can be the
b. Antitoxin cause of this disorder :
c. Antibiotic a. Schistosoma japonicum
d. Antiviral : oseltamivir b. Fasciolopsis buski
e. Antidotes c. Trichinella spiralis
d. Echinococcus granulosus
Question 16-17 are linked to the following case: e. Clonorchis sinensis
A 28-year old man presented with 8-day history
of increasing fever, malaise, headache, and
19. Refer to the previous case, the mechanism depression. From physical examination the
of edema is due to : doctor found lymphadenopathy and
a. Host-allergic response to metabolic spleenomegaly. The man is a daily farmer
wastes of parasite and a couple weeks before the symptoms
b. Reaction to the parasite as it migrates appeared he drank a glass of unpasteurized
through the tissues cow milk. The culture of blood grew a tiny
c. Reaction to embolization of eggs Gram-negative coccobacilus, catalase and
d. Immediate hypersensitivity reaction to oxidase-positive; that resemble fine grains
the adult worms in the intestine of sand. The patient was probably infected
e. Eggs damage the wall of intestine with which of the following microorganism?
a. Mycobcaterium tuberculosis
20. A 24-year-old man returned from Papua b. Salmonella typhi
Island. Five days later, he developed c. Brucella species
repeating intense chills and high fevers. d. Pasteurella pestis
These severe episodes of fever had been e. Staphylococcus aureus
occurring every other day. In between these
episodes, he had low-grade fever, myalgia, 23. A 55-year-old warden found a dead muskrat
nausea, vomiting, and diarrhea. A few hours on the bank of a stream. He picks up the
ago, he was admitted to ICU in coma. He animal and buried it. Four days later, he
became progressively somnolent and died a developed a 1.5 cm painful ulcer on the
week later. Which of the following organs index finger of his right arm, a 1 cm ulcer on
does this infectious agent initially proliferate his right forehead, and pain in his right
after entry in the infected host? axilla. Physical examination also revealed
a. Heart right axillary lymphadenopathy. Which of
b. Liver the following is working diagnosis for this
c. Brain patient?
d. Spleen a. Brucellosis
e. Renal b. Pertusis
c. Leptospirosis
d. Tularemia
21. A 61-year-old man was seen in the e. Hydatidosis
Neurology Clinic for evaluation of a
peripheral neuropathy. He was found to Question 24-25 are linked to the following case:
have dermal A volunteer man returning from a tsunami area
hyperpigmentation/depigmentation was admitted to hospital. A week before, he had
(salt/pepper) of the skin, scaly palms, and fever, headache and myalgia especially at calf.
transverse ridges on his fingernails which These symptoms resolved, but the day before
were identified as being a result of a chronic admission he became pyrexia and on
chemical intoxication. examination was found to be jaundice and to
Which of the following is the most likely have an elevated blood urea. Urine was collected
chemical agent caused that sign and and inoculated into a semisolid agar medium,
symptoms? and examined by dark ground microscopy.
a. Acetaminophen
b. Arsenic : ngumpul di rambut kuku 24. Which of the following is your working
c. Cyanide diagnosis?
d. Fluoride a. Syphillis
e. Iron b. Pertusis
c. Weil’s disease
22. A 45-year-old man from the Middle East d. Lyme disease
had fever and chills, with weight loss, e. Relapsing fever
sweats, headache, muscle pain, fatigue, and
25. Which of the following disease is the 29. A female tourist developed gastroenteritis
complication of this disease? while visiting small town in Indonesia and
a. Pulmonary hemorrhage tried some Indonesian traditional food. The
b. Myopericarditis onset of the disease is abrupt with
c. Aseptic meningitis abdominal cramps and watery diarrhea. She
d. Osteomyelitis had no fever or nausea or vomiting. The
e. Anemia symptoms have resolved within 24 hour and
no subsequent recurrences. They report the
26. A 39-year-old woman live in Manokwari, disease to district public health office. The
Papua Island developed a major seizure investigation found that one of the food
while at work. She had no history of products eaten by this tourist was
epileptic disease. A head MRI was contaminated by suspected pathogens. What
remarkable for a lesion surrounding a is the suspected pathogen may cause the
scolex. The etiologic agent would most disease above?
plausibly have been acquired by eating or a. Salmonella typhi
dinking which of the following food items? b. Shigella dysenteriae
a. Uncooked vegetables c. Enterohemorragic E. coli
b. Raw beef d. Staphylococcus aureus
c. Raw pork e. Enterotoxigenic E.coli travellers
d. Uncooked fish 30. A 23-year-old man sees his family
e. Unfiltered water physician with a sudden onset of 4-day
history of fevers, headache, retro-orbital
27. A young boy had wart-like skin lesions with pain, myalgia and rash. Physical
the appearance of strawberries in his legs. examination shows diffuse erythroderma
Later, this lesion destroys the bone of his with blanching erythema and petechial
legs (gummata), but there are no visceral formation resulting from pressure applied to
and nervous system complications. The her skin. Laboratory test reveal lymphopenia
disease is still endemic in hot tropical and thrombocytopenia. Which of the
countries, such as Indonesia. Which of the following virus is most likely responsible for
following examination is the most this infection?
appropriate to diagnosis the disease above? a. Morbili
 yaws b. Dengue
a. Widal test c. Influenza
b. IgM-IgG of Treponema d. Coxsackie
c. Dark-field microscope e. Rhinovirus
d. FTA-ABS and TP-PA test
e. ELIZA assay 31. A 48 year old lady came to the outpatient
clinic with chief complaint of fatigue and
28. A 27-year-old man develops acute severe difficulty in concentration for the last 2
encephalitis that requires hospitalization. weeks. The history taking revealed that she
Several days before he got that disease, he had a balanced diet, no history of chronic
recognizes that many pigs in his farm were disease and had been a heavy smoker for 20
died. years. On physical exam it was found that
Which of the following is most appropriated she had anaemic conjunctiva with other
natural host for the pathogen?  nipah findings was within normal limit. The
a. Arthropod laboratory examination showed that she had
b. Horse haemolytic anemia.
c. Bats Based on the information given above, what
d. Squirrel is the most likely cause for her condition?
e. Rats a. Vitamin K deficiency
b. Copper deficiency 35. A 27 year old woman in at term pregnancy
c. Vitamin E deficiency had fluctuated fever since 2 weeks ago. She
d. Iron deficiency looked pale, generalized weakness, and
e. Folate deficiency headache. She went to public health center
and was prescribed oral chloramphenicol in
32. A laboratory examination of a leprosy doses of 500 mg four times daily and
patient after 4 months MDT therapy paracetamol 500 mg three times daily. She
revealed Bacterial index 6+ and did not tell to the physician that she was
morphological index 50%. What is your pregnant. What is the expected side effect of
conclution for this result ? the drug you will find on the baby?
a. Pausibacillar leprosy patient sensitive to a. Reye syndrome
treatment b. Gray syndrome
b. Multibacillar leprosy patient sensitive to c. Down syndrome
treatment d. ADHD
c. Multibacillar leprosy patient resistent to e. Black water fever
treatment
d. Pausibacillar leprosy patient resistent to 36. A 35 year old man, a businessman, came to
treatment private doctor. He asked the physician about
e. Pausibacillar leprosy patient prone to his plan to go to Papua, which is endemic
nerve damage malaria. The physician suggested him to
take prophylactic drug. Which medication
33. A boy brought by his mother to the (WHO recommended) would be given to
emergency department of Siloam hospital this patient for prophylactic?
because of vomit & dyspnoea. According to a. Kina
his mother, her son was playing in the b. Chloroquine
backyard. On physical examination, the boy c. Doxycycline
was found to have high fever (39 ºC) and d. Kuinin
multiple stinging on the back. The most e. Primakuin
likely species of insect bites is:
a. Hymenoptera sp 37. A 40 year old man came to hospital with
b. Dermacentos andersoni: tick paralysis four times generalized tonic-clonic seizure
c. Lactodectus mactan : black widow since two week ago. There was no history of
d. Loxoxceles laeta : laba2 seizure and head trauma. A one year ago, he
e. Lytta vesicatoria : kumbang had business in Papua for six months.
During his stay in there, he often consumed
under-cooked pork. Brain MRI showed a
34. A 28 year old lady came to the Posyandu to small cystic with invaginated scolex inside.
have a nutrition consultation. Last year, she What of this following is the most likely
delivered a stillbirth baby with a spinal inside cyst?
disorder. She and her husband start planning a. Taenia eggs
to have a baby again. She asked for b. Cycticercosis cellulose
suggestion about her dietary intake before c. Cysticercosis bovis
she got pregnant. d. Taenia solium worm
What kind of diet does she need to prevent e. Taenia saginata worm
the same disorder for her future baby?
a. Dark green vegetables
b. Pasteurized milk 38. An 8-day-old infant was born with the help
c. Deep water fishes of shaman (‘dukun beranak’). The mother
d. Boiled eggs states his son became irritable since 2 days
e. Lean beef ago, and now any loud noise appears to
cause him pain, as evidenced by muscle
tightening and back arching causing his head a. Vi antigen
to nearly touch his feet. Physical b. Urease
examination reveals only a dried packing on c. Hemolysin
his umbilical cord, as is the local custom. He d. Shiga toxin
appears normal until he is stimulated by e. Pili
touch or a loud noise, and then he begins to
cry, stiffens, and arches his back. The
stiffness continues until he calms down. Of
the following, the MOST likely diagnosis is:
a. Bacterial meningitis 42. A 10-year-old girl presents with a history of
b. Botulism sore throat and difficulty breathing of 1
c. Generalized seizure day’s duration. On examination she is very
d. Tetanus ill-appearing. She has some inspiratory
e. Viral encephalitis stridor and thick white-gray material
covering her tonsils and faucial pillars, and
Question 39-40 are linked to the following case: she has swelling of her neck, no
A worried mother brings her 18-month-old son splenomegaly. The lymphocyte is normal.
to the emergency department because of a rash What is the likely diagnosis?
that developed today. She reports that he has had a. Tonsilitis
a runny nose, conjunctivitis, and diarrhea. On b. Angina Plaut Vincent
physical examination, he appears severely ill, is c. Mononucleosis infectiosa
temperature to 40°C for the last 3 days and has d. Laryngitis
diffuse 2 – 3 mm erythematous rash beginning e. Diphteria
from the hairline behind the ear and spreading to
his face and body. On buccal mucosa, there is a 43. A baby is born with a rash identical. History
red spot surrounded by white bluish appearance. taking reveals that the mother had a febrile
illness during the second trimester of
39. Of the following, the MOST likely cause of pregnancy. Examination reveals diffuse
the rash is raised purple skin lesions. There is no pallor,
a. Rubella virus jaundice, or cyanosis. The baby has cataract,
b. Coxsackievirus a 3/6 systolic heart murmur, and
c. Morbilli virus enlargement of both liver and spleen. There
d. Human herpesvirus 6 is no lymphadenopathy.
e. Parvovirus B19 Of the following, the MOST likely diagnosis
is?
40. What is the most severe complication that a. Congenital Toxoplasmosis
would happen to the patient above? b. Congenital Rubella
a. Osteomylitis c. Congenital Cytomegalovirus
b. Encephalitis d. Congenital Herpes Simplex
c. Otitis externa e. Congenital Varicella
d. Sinusitis
e. Oral thrush 44. A 40-year-old woman presented with a
raised redness plaque resemble a doughnut
41. A 48-year-old man with acute gastroenteritis with a raised border that she already had for
has step ladder night fever since 7 days ago. 1 year, It was no itchy, no pain, not easily
He also has abdominal pain, and bleeding. Sometime she feel weakness on
constipation. He usually eats uncooked her leg and her slipper left behind while
vegetables at street vendor (pedagang kaki walking. On her legs she developed an
lima). anular erythematous sharp border lesion, no
Which of the following is most likely to be a scally, shiny surface and enlargement of the
constituent of this organism? popliteal nerve. What kind of laboratory
examination results that can help us make a tests. Which of the following is the
diagnose of the case above?  kusta prevention after exposure for this case?
a. Dark field examination a. Human vaccine
b. Gram b. Health education
c. KOH examination c. Vector control
d. Ziehl neelson d. Isolation the infected people
e. Na Cl examination e. Prophylaxis antibiotics

45. A woman, recently returned from Lampung, 48. A 24 year old woman admitted to the
complains of having paroxysmal attacks of hospital with myalgia and weakness of her
chills, fever, and sweating; these attacks last right leg. She had traveled from Saudi
a day or two at a time and recur every 36 to Arabia and her immunization had never
48 h. Examination of a stained blood completed. On examination, she had normal
specimen reveals ringlike and crescent-like sensation of the right leg, with normal
forms within red blood cells. The infecting movement and strength in all other
organism most likely is extremities and a normal cranial nerve
a. Plasmodium falciparum examination. Which of the following
b. Plasmodium vivax vaccination is the best prevention for this
c. Plasmodium ovale disease?
d. Plasmodium malarie a. MMR
e. Plasmodium brasilianumi b. HBIG
c. BCG
d. DPT
e. Oral Sabin  polio

46. A 33-year-old woman ingested 10 tablets of 49. An HIV (+) patient comes to the emergency
an analgesic-antipyretic which is room with severe dehydration. The nurse
metabolized to a quinone imine. Shortly puts an IV line on him, after that the nurse
after ingestion she presented with signs of use her two hand recapping the needle, and
nausea, vomiting, and generalized malaise. got injured by that needle. The nurse is
Within 3-5 days peak hepatic dysfunction panicking. You are the doctor on duty in that
occurred and was associated with hepatic emergency room. What should you do?
necrosis. a. Press the injury site to stop the blood
Which of the following is the most likely b. Wash immediately using soap under
specific antidote that should be used to treat running tap water
this patient's over dosage? c. Use bleach or iodine to clean the injury
a. Amyl nitrite site
b. Atropine d. Use antibiotic ointment on the injury
c. BAL site
d. EDTA e. Enlarge the injury site by incision to let
e. N acetyl sistein the blood flow

47. A 27-year-old medical student was admitted


to hospital because of sudden onset fever up
to 390C and headache. Two weeks
previously he volunteers cleaning the canal
with others. On examination, he looked so 50. A 33-year-old woman at 10 weeks
yellow. Blood tests done shortly after pregnancy presents for her first prenatal
admission indicated renal function examination. Routine labs are drawn and her
abnormality and elevated liver function HBsAg is positive. Liver function tests are
normal and her Anti HBc and Anti HBs are
negative. Which of the following is the best d. Parvovirus
way to prevent neonatal infection? e. Yellow fever virus
a. Provide immune globulin to the mother.
b. Provide hepatitis B vaccine to the 54. A 45 year old man presented chronic skin
mother. lesions on both hands for the past 1 year. On
c. Perform a cesarean delivery at term. examination, his skin appears erythematous,
d. Provide hepatitis B vaccine to the crusted, fell itching, and having serous
neonate. discharge. He was working in the gold
e. Provide HBIG and the HepB vaccine to refining plant for the past 2 years, works
the neonate with potassium cyanide and never uses any
gloves. Which of the following mechanism
51. A 34-year-old G2 at 36 weeks delivers a is responsible for this illness?
growth-restricted infant with cataracts, a. Immediate respond
patent ductus arteriosus, and sensorineural b. Acute respond
deafness. She had flu-like syndrome with c. Gradual reactions : dikit2 : sulfonyl urea
rash in early pregnancy. What is the most d. Delayed reactions
likely causative agent? e. Quantal reactions : dosis berapapun
a. Parvovirus langsung kena : chloramphenicol
b. Rubella virus 55. Male, 46 years old came to you as a doctor
c. Morbili virus with chief complains shortness of breath
d. Cytomegalovirus especially at the work place. When he gets
e. Herpes simplex virus home, the symptom is missing. He also
complains that he sneezes every morning.
52. A 27-year-old female has just returned from According to him, his sister is using
a trip to Southeast Asia. In the past 24 hours, bronchodilator inhaler every day. He just
she has developed shaking, chills, and a worked at sandblasting company and expose
temperature 40o C. A blood smear reveals to dust since 2 weeks ago. Which of the
Plasmodium vivax. Which of the following following is the most appropriate to measure
agents should be used to eradicate the extra- the toxicity of hazardous material in the case
erythrocytic phase of the organism? above?
a. Primaquine a. Body Mass Index
b. Pyrimethamine b. Nutrition and Job description
c. Quinacrine c. Individual Susceptibility
d. Chloroquine d. Social pressure
e. Chologuanide e. Tissue and organs

53. A 25-year-old G3 at 39 weeks delivers a 56. The patient was a 3-year-old girl who was
small-for-gestational-age infant with seen by her pediatrician for a routine
chorioretinitis, intracranial calcifications, physical examination. Her mother was
microcephaly, jaundice, and concerned about her daughter’s poor
hepatosplenomegaly. The infant displays appetite. Physical exammmination revealed
poor feeding and tone in the nursery. The that the child was small for her age and had
patient denies eating any raw or a slightly enlarged liver. Blood was
undercooked meat and does not have any collected for a routine complete blood count,
cats living at home with her. She works as a since she had previously been slightly
nurse in the pediatric intensive care unit at anemic. Her hemoglobin level was within
the local hospital. What is the most likely the normal range; however, she did have
causative agent? eosinophilia (20%). The child had no history
a. Cytomegalovirus of travel. When questioned about pets, her
b. Epstein-Barr virus mother reported that she spent a great deal
c. Hepatitis B virus of time with her puppy. Of the following,
which manifestation of the infection (above its larvae can be found in visceral organ (lung,
case) is known to occur?  toxocara liver, gall bladder) is:
a. Creeping eruption  strongyloides a. Eosinofilia
b. Biliary cirrhosis b. Hyperinfection
c. Cholecystitis c. Retrofection
d. Pruritus ani  oxyuris d. Hypereosinofilia
vermicularis/enterobius e. Autoinfection
e. Visceral larva migrans
60. Refer to the previous case, further questioning
57. A patient complains of having nail size reveals that the woman frequently gardens in
excrement from his anus. On the stool her backyard. Of the following, which one is the
examination, parasite eggs contains a hexacanth transmitted form?
embryo with six hooklets surrounded by radially a. Fecal – anal transmission
striated spherical shell, 30 to 40 u in diameter b. Fecal – oral transmission
were found, the most likely diagnosis of this c. Fecal – cutaneous transmission
patient is : d. Direct contact with skin scales
a. Saginata taeniasis e. Intravenous drug abuse
b. Solium taeniasis
c. Cysticercosis 61. A 40-year-old man came to the clinic with
d. Oxyuriasis complaints of having fever more than one-week,
e. Ascariasis nausea, and fatique. He came back from
Samarida, Kalimantan. On blood smear
examination, there are late trophozoites forms,
58. The mother of a 4-year-old child notes that schizonts with 12-24 merozoites, and the
her child is sleeping poorly and scratching infected red cell is enlarged. Of the following,
his anal area. You suspect the child may the most likely diagnosis of this pasient is :
have pinworms. Which one of the following a. Vivax malaria
is the BEST method to make that diagnosis? b. Falciparum malaria
a. Examine the stool for the presence of c. Malariae malaria
eggs d. Ovale malaria
b. Examine the stool for the presence of e. Knowlesi malaria
larvae
c. Examine a blood smear for the presence
of microfilariae 62. A- 52- year old man is brought to the emergency
d. Examine transparent adhesive tape for department in an unconscious condition. On
the presence of eggs examination, he was death form more than one
e. Determine the titer of IgE antibody hour. On autopsy of the brain, there is cyst‘s
against the parasites fluid containing protoscoleces with hooklets and
many daughter cysts were found. The most
likely diagnosis of the disease is:
Question 59-60 are linked to the following case: a. Hydatidosis
A 30-year-old woman presents with abdominal b. Cysticercosis
pain and diarrhea of 3 day’s duration. She does not c. Cerebral malaria
complain of nausea, vomiting, or fever. She has no d. Occult filariasis
sick contacts or significant travel history. A stool e. Trichinosis
sample is obtained, which reveals rhabditiform(,
filariform: infective) larva strongyloides .
63. An-6-year-old girl sustains a large laceration
contaminated with dirt after falling from her
59. In condition below adult form of the pathogenic bike. Her mother can’t recall how many
agent can be found in whole digestive tract and doses of immunization her daughter has
received. Which of the following is the a. Observation
prevention after exposure for this case? b. Cryotherapy
a. Adult-type TT c. Antivenin : antivenom
b. TIG d. Incision and suction
c. TT and TIG e. Corticosteroids
d. HiB containing TT and TIG
e. DPT 67. A 29-year-old man has weight loss, white
plagues on the pharynx, and purple lesions
Question 64-65 are linked to the following case: on the abdomen, which on biopsy reveal
A 23-year-old man presents with extreme Kaposi sarcoma. Which of the following is
swelling of his legs and scrotum. The skin the most predictive of the patient’s
associated with the swollen areas is thick and prognosis?  HIV
scaly. The patient admits to an episode of fever a. CD4 cell count
associated with enlarged inguinal lymph nodes b. CD4/CD8 cell ratio
some time ago, but did not think much of it.  c. Degree of lympadenopathy
filariasis d. Level of HIV-1 RNA in plasma
e. Rate of decline in anti-HIV antibody
64. Which one of the following is the BEST 68. A patient come with an itchy recured fungal
method to make that diagnosis? infection on the feet and need some advice
a. Examine the stool for the presence of to be cured and some prevention measures
eggs so the diasease will not occure again, doctor
b. Examine the stool for the presence of already gave him some medication for
larvae fungal infection. What kind of advise can
c. Examine a blood smear for the presence make him healthy?
of adult worm a. Don’t eat seafood
d. Determine the titer of IgE antibody b. Use always sandals
against the parasites c. Keep the skin dry and clean
e. Examine a blood smear for the presence d. Don’t use shoes all day long
of microfilaria e. Don’t scratch, will spread the infection

69. A woman came with the complaints of being


65. According to the case above, which of the uncomfortable and hardened in the left
following is the most appropriate time to upper side abdomen. Every four days, she is
take the specimen for established the intermittently feverish. On blood smear
diagnosis? examination Plasmodium parasite +. Fever
a. Anytime symptom on the previous question referred
b. At 10.00 – 12.00 AM to :
c. At 10.00 – 12.00 PM a. Saorozoites is in the the blood
d. 30 minutes after fever b. Hypnozoites in the liver is formed
e. 120 minutes after fever c. Merozoites discharged from schizonts
enter the blood
66. At night, a 28-year-old man is brought to the d. The process of gametocytes in the blood
emergency department after his right foot e. Pigment in the parasite is formed
being bitten by an animal, within 1 hour
before admission. The patient said that he 70. A 35-year-old man as a farm worker who
heard the sound of the animal. He complains was working with pesticides is brought to
of pain in his right foot and there are two the emergency room with headache,
bitten marks, surrounded by edema and vomiting, salivation, diarrhea, muscle
erythema in the dorsal aspect of his right fasciculation, difficulty walking, and
foot. Initial management of the case difficulty speaking. His clothing has been
described should be removed, he has been washed, and he has
been given activated charcoal. What is the a. Trombositopenia
most effective remaining treatment for this b. Neutropenia
case of pesticide poisoning? c. Anemia
a. Epinephrine d. Decreased of hematocrit : arusnya naik
b. Antacid e. Leucopenia
c. Spironolactone
d. Atropine 74. A 1 year old girl came to the Puskesmas
e. Hidrochlorothiazide with swollen gum for a week. From the
history taking it was found that she was born
71. A 65-year-old-woman suddenly had flaccid normal with normal birth weight, had been
paralysis of her left leg. A few days ago, she breastfed up to now and had a balanced diet
had headache, fever, sore throat, and nausea. with high intake of cooked vegetable and
She lives with her son; daughter in law and a fruit since 6 months old. She had no history
young grandchild who’s just received a of chronic diseases. Her other physical and
routine oral vaccination. She is taking laboratory examination was normal.
immunosuppressant for her kidney’s Based on the information given above, what
transplant. Her vital signs and cranial nerve is the most likely cause of her complaint?
examination are normal. A head CT scan a. Protein deficiency due to prolonged
and lumbar MRI are also normal. breastfeeding
What is the most likely transmission of her b. Carbohydrate deficiency due to
infection? increased calorie need
a. Droplet c. Fat soluble vitamin deficiency due to
b. Air borne high vegetable/fruit intake
c. Fecal-oral d. Water soluble vitamin deficiency due to
d. Close contact cooking process
e. Animal bite e. Mineral deficiency due to competition
with high vitamin absorption

72. A pork-eating village in the highlands of 75. A 10-year-old boy who has moved to your
Papua New Guinea is reported to be practice recently has sore throats, get high
suffering from an epidemic outbreak of temperature. The child reports a runny nose,
epileptiform seizures. You have been sent to mild cough, and abdominal pain. Findings
investigate. One of the first things you on physical examination include a
should investigate is? temperature of 38°C, and vesicular lesions
a. The level of Balantidium coli in swine on the soft palate, dorsal and palmar of
stool hands. There is no cervical adenopathy or
b. The practice of consuming raw deceased rash.
human brains Of the following, the MOST likely diagnosis
c. The presence of taenia eggs in the is  hfmd
drinking water a. Adenovirus infection
d. The presence of tropozoites in the b. Coxsackie virus infection
human blood c. Mononucleosis
e. The quantity of culidae in the village d. Sinusitis
e. Streptococcal pharyngitis
73. A 34-year-old woman complained a sudden
onset of high fever for 4 days with nausea,
vomiting, headache, muscle ache. There 76. A 19-year-old woman comes to Emergency
were petechiae on examination. Which of Department with signs and symptoms of
the following laboratory result is most poisoning of an agent include nausea,
appropriate with the patient’s diagnosis?  vomiting, abdominal cramps, diarrhea,
dhf excessive salivation, headache, giddiness,
rhinorrhea, tightness in chest, pin-point
pupils, mental confusion, and muscle
twitching. Which of the following is the 80. An outbreak investigation note many
most likely agent caused those symptoms? costumer from a café were admitted to the
a. Acetaminophen hospital. They have fever; nausea-vomiting,
b. Barbiturate constipated or diarrhea, weakness and
c. Carbamate altered mental status. Rose spots are seen on
d. Cyanide the trunk. Blood cultures from patients grow
e. Opiate a non-lactose-fermenting gram-negative rod.
These people are infected from the carrier
77. A 23-year-old G1 with a history of a flulike person
illness, fever, myalgias, and In which of the following sites are bacteria
lymphadenopathy during her early third most likely to be found in carrier person?
trimester delivers a growth-restricted infant a. Blood
with seizures, intracranial calcifications, b. Kidney
hepatosplenomegaly, jaundice, and anemia. c. Liver
She has many pets in her house. What is the d. Intestine
most likely causative agent? torch e. Gall bladder
a. Trepanema pallidum
b. Trichomonas vaginslis TROPICAL MEDICINE – MCQ 2009
c. Toxocara canis  ocular dan visceral
larvae migrans 1. A young man was admitted to hospital after
d. Neissheria meningitidis increasing left arm pain and paresthesia.
e. Toxoplasma gondii Several days ago, he contacted with a dog.
His symptoms increased and were
78. A 22-year-old man who works in the accompanied by hand spasm and sweating
pediatric ward of a hospital suffers from on the right side of the face and trunk. This
malaise, sneezing, and runny nose. He patient was admitted to the hospital the day
subsequently develops a mild sore throat, after developing dysphagia, hypersalivation,
headache, and stuffy nose. The symptoms agitation, and generalized muscle twitching.
resolve within 4 days. Which virus is most Which of the following could rapidly
likely to be responsible for these symptoms? destroy the pathogenic agent of this disease?
a. Rota virus a. Infrared radiation
b. Rubella virus b. Catalase
c. Coxsackie virus c. Alkali water
d. Hepadnavirus d. Sunlight
e. Influenza virus e. Heating at 60ºC for 30 minute
79. Approximately 4 hour after eating a meal in Question 2-3 are linked to the following case:
restaurant, 3 members of a tourisms group A 65-year-old-woman suddenly had paralysis of
develop a sudden onset of nausea, vomiting, her left leg. A few days ago, she had headache,
severe abdominal cramps, and diarrhea. fever, sore throat, and nausea. She lives with her
Nobody got febrile. Which of the following son; daughter in law and a young grandchild
vaccination is should be given to the who’s just received a routine immunization. She
traveler? is taking immunosuppressant for her kidney’s
a. Shigella vaccine transplant. Her vital signs and cranial nerve
b. Salmonella vaccine examination are normal. A head CT scan and
c. Enterobacter vaccine lumbar MRI are also normal.
d. H. pylori vaccine
e. ETEC vaccine
2. What is the most likely transmission of her 6. Which is a target cell receptor for this
infection? pathogenic agent?
a. Droplet a. Silica acid on epithelial cells
b. Air borne b. Acetylcholine on neurons
c. Fecal-oral c. CD4 molecule on T lymphocytes
d. Close contact d. C3d complement on B lymphocytes
e. Animal bite e. Immunoglobulin on epithelial cells

3. Which of the following is the primary 7. Which of the following is the best specimen
pathologic effect in this case? for diagnosis in case above?
a. Persistent viremia f. LCS
b. Focal multiplication g. Blood
c. Immune complexe formation h. Urine
d. Destruction of infected cells i. Throat swab
e. Reduce of acethylcholine receptor j. Rectal swab

Question 4-5 are linked to the following case: 8. A 2 year old child presented with a history
A young boy is suffering from high fever, of febrile upper respiratory tract infection.
difficult to swallow water, hydrophobia, and His clinical conditions deteriorate with a
disorientation. On examination, there is a worsening cough and dyspneu. He was
hypoesthesia on his left arm. Four weeks ago, he admitted to the hospital. On clinical
had been bitten by a dog. The characteristics of examination, the child had obvious evidence
etiologic agent were zoonotic, single-stranded, of respiratory distress with rib retraction and
negative-sense DNA virus. an elevated respiratory rate. A chest x-ray
showed hyperinflation of both lung fields.
4. Which of the following is usefull for making What is the most appropriate specimen for
a diagnosis in this case ? laboratory diagnosis in this case?
a. Giant cell a. Nasopharyngeal washing
b. Limfosit plasma biru b. Tracheal aspirate
c. Negri bodies c. Bronchial washing
d. Inclusion bodies d. Nasal swab
e. Clue cell e. Blood

5. Which of the following is available for Question 9-10 are linked to the following case:
treating this patient? A 31 year old female underwent allogenic bone
a. Immune globulin marrow transplantation for acute myeloid
b. Live attenuated vaccine leukemia. In the second month post transplant,
c. Antiviral she developed high fever, a nonproductive
d. Antimicrobial cough and breathlessness. Physical exam
e. Antitoxin showed splenomegaly, and laboratory studies
showed anemia, leucopenia, and
Question 6-7 are linked to the following case: thrombocytopenia, and abnormal liver function
A 24 year old woman admitted to the hospital test. Her chest x-ray showed diffuse interstitial
with myalgia and weakness of her right leg. She pulmonary infiltrates. A bronchoalveolar sample
had traveled from Saudi Arabia and her performed infected kidney cells showed owl eye.
immunization had never completed. On
examination, she had normal sensation of the 9. What is the best treatment for this infection
right leg, with normal movement and strength in disease?
all other extremities and a normal cranial nerve a. Antimicrobial
examination. b. Antiviral
c. Antitoxin
d. Immunosuppressant lypmadenophaty and blisters on the penis.
e. Antifungal Which of the following pathogens is most
likely to have caused his disease?
10. This female’s illness could be most easily a. Human papilloma virus
diagnosed by which of the following tests? b. Haemophillus ducreyi
a. HSV-1 IgM antibody c. Treponema pallidum
b. CMV antigenemia d. Chlamydia trachomatis
c. HHV-6 IgM antibody e. Herpes simplex virus
d. Culture of blood in erythroblastic cells
e. Monospot test 14. A 4 years old boy with sickle cell disease is
taken to his pediatrician’s because he is pale
11. In an elementary school many of students and is suffering from repeating intense chills
suffer from sore throat, runny nose, cough, and daily high fever. A blood test shows that
fever, headache, myalgia, and fatigue. The WBC was 17.000/µl and a platelet count is
students who have not been vaccinated 53.000/µl. What description is relevant to
against this infection are offered the findings in this patient?
prophylactic treatment with certain antiviral a. Sickle cell trait decreased the patient’s
drug. What is the mechanism of action for resistant to the virus infection
this drug? b. Sickle cell trait promotes infection of
a. Blockade of nucleocapsid release erythrocytes by plasmodium
b. Blockade of viral attachment c. Virus infection promotes INF
c. Blockade of viral protein synthesis production
d. Inhibition of RNA-dependent DNA d. Virus infection targets erythrocyte
polymerase precursor cells
e. Inhibition of RNA-dependent RNA e. Bacterial infection activates leukocytes
polymerase cells

12. A 28 year old female nurse became ill after 15. A 26-year-old nulliparous woman, in her
taking care of a patient with pneumonia 2 seventh month of pregnancy complains of a
days ago and was admitted to the hospital 7 days history of fever especially at night.
with fever and sore throat. She was placed in As she recall, 2 weeks ago she ate gado-
respiratory droplet isolation and quickly gado in coastal area when she out for duty
developed respiratory failure, requiring for several days. She also had diarrhea and
mechanical ventilation. In 10 days, nine abdominal discomfort. On physical
other patients in the same ward developed examination reveals blood pressure 110/70
the same symptoms. Which of the following mmHg, heart rate 60 ×/min, RR 20x/min,
laboratory test is most appropriate? temp. 39.1°C. Tongue is coated and tremor.
a. Blood culture to identify S.pneumoniae Fetal USG shows normal.
b. Urinary antigen testing for Legionella What is the most likely etiologic factor
pneumophila above?
c. Virus culture to identify influenza A a. Salmonella typhi
d. Serology to demonstrate antibodies to b. Plasmodium falciparum
identify SARS c. Vibrio cholera
e. PCR of nasopharyngeal secretion to d. Hepatitis A
identify RSV e. Escherichia coli

13. A young man presented with headache, low


grade fever, and painful blisters on the 16. A 4-year-old child presents with fever,
penis. He admitted to having had cough, conjunctivitis, coryza, photophobia,
unprotected sex in the past month. Physical posterior cervical adenopathy. Red lesions
examination revealed inguinal with a white center are present on the buccal
mucosa. A generalized erythematous rash is a. Neoplasma
also noted. What is the most likely b. Aneurisma
diagnosis? c. Infarction
a. Rubella d. Abcess
b. Kawasaki disease e. Hemorrhage
c. Adenovirus infection
d. Rubeola Question 20-21 are linked to the following case:
e. Varicella
A 5 years old boy came to the clinic with
17. A 35-year-old man as a farm worker who recurrent blanking out at school for 1 month. He
was working with pesticides is brought to had recurrent episodes in which he abruptly
the emergency room with headache, stops all activity for about 10 seconds, followed
vomiting, salivation, diarrhea, muscle by a rapid return to full consciousness. After the
fasciculation, difficulty walking, and episode patient resumes whatever activity he
difficulty speaking. His clothing has been was previously engaged with no awareness that
removed, he has been washed, and he has anything has occurred.
been given activated charcoal. What is the 20. What is the most likely diagnosis?
most effective remaining treatment for this a. Simple partial seizure
case of pesticide poisoning? b. Complex partial seizure
a. Epinephrine c. Absence seizure
b. Antacid d. Atonic seizure
c. Spironolactone e. Secondarily generalized seizure
d. Atropine
e. Hidrochlorothiazide 21. What is the most appropriate antiepileptic
drug for case above?
18. Mr. Farid is a diabetic. He read in a book a. Carbamazepine
that the main source of the brain fuel is b. Valproic acid
glucose. His doctor said that insulin in his c. Benzodiazepine
body is not enough for entering glucose to d. Lamotrigine
the cell. So he worries about the work of his e. Ethosuximide
brain. You know that the brain is protected
from high molecule particle by blood-brain Question 22-23 are linked to the following case:
barrier (BBB). What do you think?
a. The main brain fuel is not glucose A 70 years old man presents to ER with history
b. Glucose is not big enough to be block of seizure. His wife awakened at 05.00 by an
by blood-brain barrier odd gurgling noise. Her husband’s head was
c. Glucose enter the brain through the deviated to the left and his left arm was
leakage part of the BBB stiffened. After a few moments he had
d. There is special transporter for glucose generalized body jerking and was unresponsive.
to enter the brain Event lasted 2 minutes but stopped
e. Glucose in the only exception for blood spontaneously. His wife said he seemed drowsy
brain barrier and confused. There was no history of prior
seizure. On ER, he was conscious, BP was
19. A 45 year old man has chronic otitis media 140/95 mmHg, HR 96 x/min, RR 20 x/min,
since 2 years ago. He comes to Siloam Temperature 36, 5 C. The laboratory
hospital because he suffers headache since a examination revealed Random Blood Glucose
week ago. The doctor makes some was 610 mg/dl.
examinations also radiographic examination
on his head. The doctor fined a round mass 22. What is the most likely diagnosis?
with 3 cm diameter. Which of the most a. Pseudoseizure
likely cause of the mass? b. Primary epilepsy
c. Secondary epilepsy days and several greenish vomiting. His
d. Acute provoked seizure vital signs are deteriorating. On physical
e. Generalized seizure examination, the boy has abdominal
tenderness and the doctor finds palpable
23. What is your treatment for case above? abdominal mass in the hypogastric area.
a. Diazepam intravenous Prior to this, he has recurring abdominal
b. Oral phenytoin pain and bloody stool for several weeks, has
c. Ringer lactate infusion history of passage of worm through his
d. Insulin subcutan rectum, and has been treated with
e. Dextrose bolus intravenous antihelminth.The boy and his family live in
a suburb area and he loves to play marble
with his friends. His nails are dirty and his
mother told the doctor that he often forget to
wash his hands before eating. What is urgent
24. A 5-year-old girl brought by her mother to a further study the doctor should take to make
clinic because of a-two-day fever and sore the diagnosis?
throat. On mouth examination, the doctor a. Stool examination
finds a leathery grayish white membrane b. Complete blood count
adheres to pharyngeal walls and tonsillar c. Blood glucose level test
pillars which spread asymmetrically onto the d. Plain abdominal radiography
soft palate and uvula. What treatment should e. Serologic test
be given to the patient?
A. Antibiotic and antiviral 27. One member of the NGO (Non-
B. Antiviral and antifungi Governmental Organization) team from
C. Antitoxin and antifungi Indonesia has been reported dead in Mexico
D. Antibiotic and antitoxin because of a spider’s bite. The possible
E. antiviral and antitoxin cause of this disorder is:
a. Erucism
25. A 22-year-old woman who just got home b. Lepidopterism
from a 2-week voluntary work in rural c. Tick paralysis
Papua comes to a clinic with history of d. Arachnidism
bloody and slimy diarrhea since her last five e. Delusional parasitosis
days in Papua. She experiences more than
five bowel movements within each day. She
also experiences stomach cramp and painful 28. A boy brought by his mother to the
passage of stool. She reports no fever or any emergency department of Siloam hospital
other complains. There is a slight lower because of vomit & dyspnoea. According to
quadrant abdominal tenderness on her his mother, her son was playing in the
physical examination. She is found to be a backyard. On physical examination, the boy
little bit dehydrated. What is the most likely was found to have high fever (39 ºC) and
organism which causes her condition? multiple stinging on the back. The most
a. Entamoeba histolityca likely species of insect bites is:
b. Vibrio cholerae f. Hymenoptera sp
c. Rotavirus g. Dermacentos andersoni
d. Salmonella typhi h. Lactodectus mactan
e. Escherichia coli i. Loxoxceles laeta
j. Lytta vesicatoria
26. A 7-year-old boy presents in emergency unit
with colicky abdominal pain since two days Question 29-30 are linked to the following case:
ago. He also has fever and appears lethargic.
He has experienced constipation for four
29. A 10 years child came with the complaints following, which parasite can be the cause
of diarrhea, occasionally with blood & of this disorder :
mucus. 2 days ago, the patient complained a. Plasmodium falciparum
about the swelling in the anus. On physical b. Plasmodium vivax
examination was found prolapsed of the c. Plasmodium malariae
rectum. On stool examination was found d. Plasmodium ovale
parasite eggs. The most likely cause is the e. Plasmodium cynomolgi
worm of species:
a. Ascaris lumbricoides 34. An employee who works in Bogor has a task
b. Necator americanus in Samarinda for 3 weeks. What is the most
c. Angilostoma duodenale possible drug (s) should be given for this
d. Trichuris trichiura condition?
e. Enterobius vermicularis a. Chloroquine+ primaquine
b. Sulfadoksin-pirimetamin
30. Refer to the case above, the habitat of this c. Atersunate +amodiaquine
worm is located on d. Kina+tetracycline
a. Liver e. Doxycycline
b. Lung
c. Caecum 35. The mother of a 4-year-old child notes that
d. Small bowel her child is sleeping poorly and scratching
e. Stomach his anal area. You suspect the child may
have pinworms. Which one of the following
Question 31-32 are linked to the following case: is the BEST method to make that diagnosis?
a. Examine the stool for the presence of
A 35-year-old man was brought to the eggs
emergency department of hospital with seizure. b. Examine the stool for the presence of
On cerebro spinal fluid (CSF) examination larvae
shows neutrophils, red blood cells, and amoebae c. Examine a blood smear for the presence
form on wet microscopy. of microfilaria
d. Examine transparent adhesive tape for
31. Which of the following, which parasite can the presence of eggs
be the cause of this disorder : e. Determine the titer of IgE antibody
a. Naegleria fowleri against the parasites
b. Entamoeba histolytica
c. Acanthamoeba castellani 36. A 23-year-old man presents with extreme
d. Entamoeba coli swelling of his legs and scrotum. The skin
e. Plasmodium falciparum associated with the swollen areas is thick
and scaly. The patient admits to an episode
32. What is the transmission of this disease? of fever associated with enlarged inguinal
a. Ingestion of infected snails lymph nodes some time ago, but did not
b. Food & water contaminated with think much of it. Which one of the following
parasite is the BEST method to make that diagnosis?
c. Mosquito bites a. Examine the stool for the presence of
d. Sexual contact eggs
e. Intranasal infection while swimming in b. Examine the stool for the presence of
the warm fresh water larvae
33. A woman came with the complaints of being c. Examine a blood smear for the presence
uncomfortable and hardened in the left of microfilaria
upper side abdomen. Every four days, she is d. Examine a blood smear for the presence
intermittently feverish. On blood smear of adult worm
examination Plasmodium parasite +. Of the
e. Determine the titer of IgE antibody d. The process of gametocytes in the blood
against the parasites e. Pigment in the parasite is formed

37. A patient came to the clinic with portal Question 41-42 are linked to the following case:
hypertension syndromes. From the interview, the
patient was known to consume raw fish A 30-year-old man, having fever for 3 days,
regularly. Of the following, the most likely with falciparum rings +++ was given CQ3
cause of disease is : (chloroquine for 3 days). After 3 days, the
a. Paragonimiasis patient is still feverish, blood smear examination
b. Clonorchiasis shows falciparum rings ++++.
c. Fasciolopsiasis
d. Echinococcosis 41. What is the possible response of CQ?
e. Fascioliasis a. Sensitivity (S)
b. R I resistance, early recrudescence
c. R I resistance, delayed recrudescence
38. A 40-year-old patient comes with the complaints d. R II resistance
of uncomfortable stomach & flatulence. From e. R III resistance
history, the patient was known live in the valley
of Napu, Sulawesi and he had a fever frequently. 42. Refer to the previous case above, what is the
On physical examination, his liver was found treatment of choice?
enlarged. What is the most likely caused of the a. Primakuin
disease? b. Amodiakuin
a. Schistosoma haematobium c. Meflokuin
b. Schistosoma mansoni d. Sulfadoksin pirimetamin
c. Schistosoma rodentatum e. Artemisinin
d. Schistosoma japonicum 43. 52- year old man is brought to the
e. Schistosoma mekongi emergency department in an unconscious
condition. On examination, he was death
Question 39-40 are linked to the following case: form more than one hour. On autopsy of the
brain, there is cyst‘s fluid containing
A 40-year-old man came to the clinic with protoscoleces with hooklets and many
complaints of having fever more than one-week, daughter cysts were found. What is the most
nausea, and fatigue. He came back from Timika, likely diagnosis of this disease?
Papua. On blood smear examination, there are late a. Hydatidosis
trophozoites forms, schizonts with 12-24 b. Cysticercosis
merozoites, and the infected red cell is enlarged. c. Cerebral malaria
d. Hstoplasmosis
39. Of the following, the most likely caused of e. Toxoplasmosis
parasite species is :
a. Plasmodium vivax 44. A 27 year old women suffering fever for 2
b. Plasmodium falciparum day. She brought to emergency department
c. Plasmodium malariae unconsciously and she had general seizure 2
d. Plasmodium ovale hour ago. On physical examination she is
e. Plasmodium cynomolgi comatous, GCS (Glasgow Coma Scale) : 3;
BP 90/60 mmHg; t 41 °C. Lab. finding: Hb
40. Fever symptom on the previous question 10, 3 gr%; WBC 5800/mm3; platelet
referred to : 39.000/mm3; there is schizon form on blood
a. Sporozoites is in the blood smear. Why the pathogeniv agent of this
b. Hypnozoites in the liver is formed disease becoming the most dangerous species?
c. Merozoites discharged from schizonts a. It make long term relaps
enter the blood b. Intermittent fever
c. Gametocyte formation take place in bike. Her mother can’t recall how many
visceral organ doses of tetanus toxoid her daughter has
d. Can make capillary obstruction in Central received. Management of tetanus
Nervous System by parasite contain prophylaxis in this situation of unknown
erythrocyte history of prior doses of tetanus toxoid
e. There are many drug resistance against P. includes:
falciparum a. Adult-type TT
b. TIG
45. A 35-year old man working on pig farm is c. TT and TIG
brought by his employer to the emergency d. HiB containing TT and TIG
department of Siloam hospital because of e. DPT
seizure and bilateral lower extremity
weakness. CT scan of the head reveals several 48. If this patient developed seizures, what
calcified regions. Laboratory examinations substance has responsible in this patient?
show WBC count of 10.800/mm3, with a. Tetanospasmin
eosinophils. Brain tissue biopsy reveals scolex b. Tetanolisin
with hooklets. Which one is the treatment of c. Adenylate cyclase toxin
choice for this case? d. Fillamentous hemagglutinin
a. Sulfadiazine-pirimetamin e. Pertactin
b. Artemisinin
c. Albendazol Question 49-50 are linked to the following case:
d. Metronidazol
e. Itrakonazol A 3-year-old boy was brought to emergency unit
in Tangerang Hospital with stridor, dyspnea, and
46. A female tourist developed gastroenteritis “croupy” cough.
while visiting small town in Indonesia and
tried some Indonesian traditional food. The 49. What is the most likely pathogenic agent
onset of the disease is abrupt with that causes the patient’s disease?
abdominal cramps and watery diarrhea. She a. Bordetella pertussis
had no fever or nausea or vomiting. The b. Haemophilus influenzae
symptoms have resolved within 24 hour and c. Coxsackievirus
no subsequent recurrences. They report the d. B. Parapertussis
disease to district public health office. The e. C. diphtheriae
investigation found that one of the food
products eaten by this tourist was 50. What is the laboratory test to confirm the
contaminated by suspected pathogens. What diagnosis in this patient?
is the suspected pathogen may cause the a. Blood culture
disease above? b. Throat swab culture
c. Chest X-ray
f. Salmonella typhi d. Measurement of IgG antibody serum
g. Shigella dysenteriae e. Urine culture
h. Enterohemorragic E. coli
i. Staphylococcus aureus
j. Enterotoxigenic E.coli Question 51-52 are linked to the following case:

A 2-year-old boy was brought to emergency unit


Balaraja Hospital with difficulty in breathing,
Question 47-48 are linked to the following case: choking, gasping, and whoop cough.

47. An-8-year-old girl sustains a large laceration 51. What is your most likely diagnosis?
contaminated with dirt after falling from her a. Bordetella pertussis infection
b. Haemophilus influenzae infection 55. A 2 years old boy brought to
c. Coxsackievirus infection dermatovenereology clinic because of
d. Adenovirus infection itching all of the body. He complains itching
e. Corynebacterium diphtheriae infection on palm, side of finger, buttock, and
abdomen. Clinical manifestations are
52. In this case all the family members should papule, vesicle, pustule, erotion, and
get the prophylaxis antibiotics as follow: excoriation especially at finger web, arm,
a. Amoxicillin 30-50 mg/kg/day p.o. for 14 abdomen, side of hand and feet, itchy
days especially at night.His neighbours have
b. Amoxicillin 30-50 mg/kg/day p.o. for 7 same problem.He usually play together with
days his neighbours.What is the diagnose this
c. Erithromycin 40-50 mg/kg/day p.o. for patients?
14 days a. Pediculosis corporis
d. Erithromycin 40-50 mg/kg/day p.o. for 7 b. Scabies
days c. Tinea corporis
e. Tetrasiklin 25-50 mg/kg/day p.o. for 7 d. Dermatitis atopik
days e. Psoriasis

Question 53-54 are linked to the following case: 56. What is the best treatment to case above?
a. Sulfur
Male, 46 years old came to you as a doctor with b. Gamexane
chief complains shortness of breath especially at c. Crotamiton
the work place. When he gets home, the d. Lindane
symptom is missing. He also complains that he e. Permethrine
sneezes every morning. According to him, his
sister is using bronchodilator inhaler every day. 57. A 70 years old women, come to
He just worked at sandblasting company and dermatovenereology clinic because she
expose to dust since 2 weeks ago. complains pain, edema on her left eyelid and
there is some vesicle in her left forehead.
53. What are the most possible causes of his She complain pain on her forehead 2 days
disease? ago, but vesicle come out 1 day ago.She
a. Wood sometimes fells headache and fever. Clinical
b. Quartz manifestation are grouped vesicle, eritema
c. Tobacco smoke base at her left face.What is the diagnose of
d. Coal this patient?
e. Amonia a. Varicella
b. Herpes zoster
54. The doctor gave him sick-leave certificate c. Herpes simpleks
for 2 days and asks the patient to come back d. Dermatitis venenata
tomorrow to perform spirometry. What is e. Dermatitis iritan
the result of spirometry do you expect?
a. FEV1 decreased 58. A 43 years old women, come to
b. FVC decreased dermatovenereology clinic because she
c. FEV1 and FVC decreased complains vesicle at genital, and pain, for
d. Normal the first time. Her husband has same
e. MVV increased problem 2 days ago,but it was cured. She is
difficult to walk because of pain on her
genital and low fever. What is the most
Question 55-56 are linked to the following case: appropiate treatment for this patient?
a. Acyclovir 5 x 200 mg, for 7 days
b. Famciclovir 3 x 500 mg,for 7 days
c. Valacyclovir 2 x 250 mg, for 7 days b. Genetic factors
d. Acyclovir 2 x 400 mg for 7 days c. Cultural factors
e. Glucocorticoid topical d. Tropical factors
e. Synergisms on genetic, cultural, and
Question 59-60 are linked to the following case: immune factors

59. A 34 years old man, come to 63. Tropical disease category III is group of
dermatovenereology clinic, complain itchy tropical disease which control strategy
on his sole. He is a army, that usually proven effective, disease burden falling, and
exercise at dirty land. Clinical elimination planned. What are the examples
manifestations are burrow 3 cm on his sole, of disease of this category?
erithema, and papule. What is the treatment a. Leprosy, lymphatic filariasis
for patient above? b. Malaria, tuberculosis, dengue,
a. Lindane schistosomiasis
b. Gamexane c. Ebola, avian influenza, SAR
c. Albendazole d. Protein calorie malnutrition,
d. Pirantel pamoat pesticide intoxication
e. Antibiotika e. Hepatitis B, pertusis, poliomyelitis

60. A 30 year old women, come to


dermatovenereology clinic, because she
complains many vesicle on her body, low
fever, muscle pain, and sore throat . She was 64. A 24-year-old migrant farm worker is
pregnant 3 months. What is the diagnose of rushed to a nearby emergency room after an
her illness? accidental exposure to parathion. He is
a. Varicella in respiratory distress and is
b. Herpes Zoster bradycardic. Which of the following
c. Herpes simplex drugs can be given to increase the
d. Moluscum Contagiosum activity of his acetylcholinesterase?
e. Rubella a. Atropine
b. Deferoxamine
61. In epidemiological perspective, what is the c. Dimercaprol
tropical disease? d. Physostigmine
a. Infectious disease in the tropics e. Pralidoxime
b. Any communicable disease
c. Disease which deaths occur in 65. A migrant worker presents to the emergency
children under five room in respiratory distress. He had been
d. Disease which deaths occur in spraying parathion in the fields for several
productive age days, and today he began to feel sweaty and
e. Primarily found in the developing dizzy. By the time he got to the hospital he
world or profoundly impact the was drooling, gasping, and becoming
health of people living in the tropics agitated. These symptoms are due to the
actions of parathion on which of the
62. Conditions that contribute to the risk for following?
becoming infected with tropical diseases a. Acetylcholinesterase
agent: b. Muscarinicreceptors
a. Biological factors related to c. Neuronallipidbilayer
population density, rural vs. urban d. Nicotinicreceptors
living, nutritional status, climate and e. Voltage-gated sodium channels
other environmental factors, as well
as socioeconomic circumstances
66. A 25-year-old woman is despondent that her Based on this information, he was likely to
husband left her for another woman. She be classified in which condition below?
attempts suicide by ingesting 25 tablets of a. Marasmus
extra strength acetaminophen (=650 mg). b. Kwashiorkor
Her mother finds her and the empty bottle a c. Marasmic-kwashiorkor
couple of hours later and immediately rushes d. Energy malnutrition
her to the emergency room. Which of the e. Protein malnutrition
following drugs will most likely be given to
this patient? 69. A one year old boy was diagnosed with
a. Acetylcysteine measles since a week ago. This morning, his
b. Atropine mother noticed that he also suffered for
c. Penicillamine swollen reddish gums that signified as
d. Pralidoxime scurvy.
e. Protamine What is the most likely mechanism that
underlines his recent condition?
67. A 46-year-old woman comes to a hospital to a. Impaired absorption of micronutrients
have several bunions removed from her right b. Altered micronutrient metabolism
foot. She chooses conscious sedation rather c. Depletion of micronutrient stores
than general anesthesia for this procedure. d. Micronutrient redistribution
She is given intravenous midazolam to e. Negative micronutrient balance
supplement the local anesthetics that are
injected into her foot. Midway through the 70. An 18 year old girl came to obstetric and
surgery, she suddenly becomes agitated, gynecologic outpatient clinic with chief
combative, and exhibits involuntary complaint no menstrual cycle for 2 months.
movements. The anesthesiologist determines She also complained for losing weight 5 kg
that she is having a paradoxical reaction to in the last 3 months. From the history taking
the midazolam. Which of the following is found that she was treated as a tuberculosis
the most appropriate drug for counteract that patient since 4 months ago.
midazolam? What is the most likely underlying
a. Flumazenil mechanism that affects her nutritional status?
b. Glucagon a. Increased gluconeogenesis
c. Naloxone b. Increased vitamin excretion
d. Nitrite c. Increased resting energy metabolism
e. Protamine d. Increased carbohydrate stores
e. Increased protein anabolic

71. A 53-year-old male farmer presents to the


primary health care complaining about a
mass on his arm. He noticed some mildly
itchy red bumps on his arm since a week
68. A 2 year old boy came to the Puskesmas before. They started to blister a day or two
with ulcer on his left hand. Through the later and then ruptured. During this time he
history taking it was revealed that he had not had a low-grade fever. Futher questioning
been well fed in this past year since his reveals he has had no ill contacts and never
mother died. From his physical examination had anything like this before. He has cows,
it was found that his BMI (body mass index) horses, goats, sheep, and chickens on his
was below 5th percentile for boys of his age. farm. On examination of his right upper
He was lethargy and looked pale with a face arm, you find a 4, 5 cm circular black eschar
appearance like a thin old person. His ribs surrounded by several blisters and edema.
were very prominent. He has tender axillaries lymph node
enlargement. A gram stain of fluid drained
from a vesicle and a biopsy from the eschar dorsalis pedis and posterior tibial artery.
both show chains of gram-positive bacilli on What is the next management of the case
microscopy. What organism is the likely described should be?
cause of this disease? a. Wide excision
a. Bacillus anthracis b. Fasciotomy
b. Bacteriodes fragilis c. Antivenin
c. Borrelia burgdorferi d. Incision and suction
d. Campylobacter jejuni e. Corticosteroids
e. Yersinia enterocolitica

SOAL UJIAN TROPIAL MEDICINE


2007
Question 72-75 are linked to the following case:

At night, a 28-year-old man is brought to the


emergency department after his right foot being 1. Putri…kerjasama nya please…. Demi kalian
bitten by an animal, within 1 hour before juga kok
admission. The patient said that he heard the
2. Putri…kerjasama nya please…. Demi kalian
sound of the animal. He complains of pain in his
juga kok
right foot and there are two bitten marks,
surrounded by edema and erythema in the dorsal 3. Anak dengan panas tinggi, myalgia, dan
aspect of his right foot. gejala flu like syndrome lain, ada susah
bernapas. Sodaranya sblm nya juga kaya gt n
72. What is the most likely animal that has udah meninggal. Apa spesimen paling baik buat
bitten the patient? menegakkan diagnosa?
a. Cottonmouth snake ɑ. Saliva
b. Copperhead snake B. Blood
c. Rattlesnake C. Stool
d. Latrodectus mactans D. Lung biopsi
e. Centruroides exilicauda E. Nasofaring swab
73. Initial management of the case described
should be
a. Observation 4. Pertama2 flu like illness, batuk ga
b. Cryotherapy sembuh2,vomiting, hoarseness,
c. Antivenin Answer: pertusis
d. Incision and suction
e. Corticosteroids
5. bitten by dog become hydrofobia,
74. The next step in the management of the
hypersalivation, spasm, and sweating.
patient should be:
a. Wide excision a. giant cell
b. Fasciotomy
c. Antivenin b. limfosit plasma
d. Incision and suction
e. Corticosteroids c. negri bodies

75. After 12 hours, the patient complains of d. clue cell


severe pain especially when he moves his
right leg and his right foot becomes pallor
and cold. There are no pulse in the right
6. kesalahan banker…kehilangan data.. maap c. Renal biopsy
yeii d. Stool exam
e. CT-Scan

A 32 year old worker in a farm having painless


dermal papule on right hand since 5 days ago. 10. A 53 y.o. farmer experiencing sub febris,
The papule is getting bigger, and the skin abdominal pain, diarrhea. In his skin there is a
become black. He has tender axilaries lymph black eschar surrounded with vesicle and there is
node enlargement. edema near his mouth. A week ago he notices
that his cattle were sick, but his still eat half
Anthrax cook cattle meat. What is the organism?

a. Bacteroides fragilis
7. Which of the following virulance factor is b. Leptospira interrogans
most likely to be involve in pathogenesis of
illness? c. Bacillus antrhacis

a. exotoxin d. Pasteurilla pestis

b. endotoxin e. Borellia brugdorferi

c. alpha hemolysin

d. lipopolysaccharide

e. antiphagotic factor 11. a women and recently return from Lapung


complaining having paroxymal attack chill,
fever and sweat. These attack last a day 2 days at
time and recur every 36 – 48 jam. Which the ost
8. A 45 years old man was seen in the appropriate time to take a blood specimen?
dermatology clinic because of some nodules on
his arm&body and sometimes feel pain with a. anytime
pressure. On examinations, we found
erythematous nodules, smooth and shiny, diffuse b. 10 – 12 a
inflitrate with ill define border, no fluctuation or
erosion. There were pain on his elbow with c. 10 – 12 pm
nerve enlargement. What laboratorium test
would you do to confirm the clinical diagnosis? d. 30 min after fever
a. TPHA & VDRL e 120 min after fever
b. Gram preparation
c. KOH preparation
d. Skin scrapping
e. Skin slit smear 12. seorang ibu minum 10 tablet hypnotic 30
menit yang lalu sebelum masuk rumah sakit.
gejala yang tampak : hypotension tachypneu,
tachycardi dan tidak sadar )unconsiusnes.
9. 40 yo patient complaint uncomfortable tindakan sebagai dokter yang yang pertama
stomach and flatulence. Live in the valley of kali dilakukan adalah ..
Napu, Sulawesi and he had a fever frequently. a. apomorphine
PE: liver enlarged. Test buat confirm? b.
a. Blood c. infusion natrium
b. Liver biopsy
d. chancoal dgn NGT e. Antidotes
e.hemodyalisis

16. A 28 years old ,an comes withu high fever


13. Outbreak TB in a dormitory, with the same for 8 days, malaise, headache and constipations.
source of infection, based on RFLP analysis. He denies any vaccines before. Temperature 39
The most appropriate health intervention? C. Blood pressure 130/90 pulse 72x/min. on
a. Rehabilitation Physical examination there are
b. Prevention hepatosplenomegaly and faint erythomatous
C. Curative measles.
D. Medication
E. Environmental modification what are the most likely pathogen?

a.vibrio cholera

14. there is rna virus cause deathly outbreak of b. Shigiella dysentry


hemoragic disease. Patient have feer, muscle
pain, headach, followed by abdominal pain. c. salmonella typhi
Rash with internal n external bleeding. Hospital d. entero hystolytica
staff is infected. Virus are highly virulent
transmited by: e. Giardia lambia
a. blood n body fluid

b. mosquito bite 17. pasien 28 tahun demam 8 hari ada ciri2


sebagai berikut : erythematousmacula, myalgia,
c. transmission to human from rodent excrata headache, vaksin tidak lengkap.
d. aerosol transmission a. typhii
e. ……….. b.dysentri

c. cholera
15. a 31 years old man with complaints of high d. giardia lambia
fever, cough, shortness of breath. 2 hours before
admission he experienced right sided chest pain e. entamoeba hystolitika
when took a deep breath and cough. Chest
radiography shows diffused bilateral interstitial
pulmonary infiltrate. Analysis of blood gas
showed PO2= 60 and 91% HB saturation. 18. Men, 44 tahun dari Jakarta pergi berbisnis ke
Hematology serum and liver are normal. He Kalimantan selatan. Lalu disana dia pergi ke ER.
lives near poultry. Patho agent are new viral.
Sign and symptom = diare and abdominal
what are the prevention for prophylaxis?
pain
a. vaccines
PE = Pitting edema
b. Antitoxin abdominal and leg

c. Antibiotik Lab culture = (-) enteric pathogen

d. Antiviral Blood = eosinophilia


Stool = large ellipsoidal, A.acetaminophen
operkuled egg with thin and transparent sheet. B.arsenic
Tekena infeksi apakah bapak ini? C.cyanide
D.flouride
Schistosoma Japonicum E.iron
Fasiolopsis Buski

Tricinella Spiralis 22. Chandra…kerjasama nya please…. Demi


kalian juga kok
Echinococcus granulosus

Clornonsis sinonsis
23. a 55 year old man from middle east had
fever and chill with weight loss, sweats,
19. bagaimana mekanisme terjadinya edema headache, muscle pain, fatique, depression. The
pada cerita diatas? man is daily farmer and a couple week before
symptoms appeared he drank glass
a. host-allergic reaction because of the parasite unpasteurized raw milk. The culture of blood
tiny gram (-) cocobacill, catalase(+) and oxidase
b. the larva penetrate the wall (+). Infected organism?
c. hypersensitivity reaction because adult worm A. M. tuberculosis
in GI
B. S. typhii
d. there are accumulation of egg in GI
C. Brucell species
e……..
D. Pasteurella pestis

E. S. aureus
20. Pulang dari papua. %hari kemudian ada
intense chill fever. demamnya dengan always 1
day after. between those period, having low
grade fever, myalgia, nausea, vomitting. datang 24. edo…kerjasama nya please…. Demi kalian
ke klinik , 1jam yang lalu masuk ke ICU- juga kok
agitated, ga lama mati. Organism yang masuk ke 25. Komplikasi plg sring ???
host ini berproliferation dmn? a. Pulmo hemoragik
a.brain
b.liver B. m miopericarditis
c.heart
d.spleen c. encephalitisa

d. Osteomyelitis

21. 61yr old man,go to the neurolog clinic untuk e. Anemia


evaluasi peripheral neuropathy. He was found to
have dermal
hyperpigmentation/depigmentation(salt/pepper)
of the skin,scaly palms and transverse ridges on 26. A 39 years old man, live in Manokwari
hus fingernail which were identified as being a papua, developed a major seizure while at work,
result of a chronic chemical intoxication. no history of epilepsy. Head MRI showed lesion
What chemical agent? surrounding a scolex. The etiolohic agent would
most possibly have been acquired by eat/drink:
A. Uncooked vegetables
B. Raw beef 30. 23 years old male. 4 days history of fever,
C. Raw pork headache, retroorbital pain, myalgia and rash.
D. Uncooked fish PE diffuse er
E. Unfiltered water
resulting from a pressure applied to her skin.

morbili
27. anak cowo ada lesi di kulit seperti
strawberry, gummata. Exam apa yang paling dengue
cocok untuk diagnosis? influenza
a. widal test coxackie
b. IgM-IgG treponema rhinovirus
c. dark field

d. FTA-ABS 31. a 40 year old lady came to the outpatient


e. ELISA clinic with a chief complaint of fatigue and
difficulty in concentration for the last two
weeks. HT revealed that she had a balance diet,
no history of chronic disease and had been a
28. A 27 year old man come with acute severe heavy smokers for 20 years. On PE it was found
encephalitis that require hospitalization. Several that she had anaemic conjunctiva with other
days before he got that disease he recognize findings was within normal limits. Lab showed
many pigs in his farm were died. Which of the that she had haemolytic anemia. What is the
following is most appropriated natural host for most likely cause for her condition?
the pathogen?
A. Arthropod a. vitamin K deficiency
B. Horse
C. Bats b. copper deficiency
D. Squirel c. vitamin E deficiency
E. Rats
d. iron deficiency

e. folate deficiency
29. female tourist gastroenteritis ke Indonesia
makan Indonesia food. Abdominal cramp and
watery diarrhea. No fever or nausea or vomiting.
Symptom resolve 24 hours no recurrent. 32. a lab examinaton of a leprosy patient after 4
Pathogen? months MDT therapy revealed bacterial index
6+ morphological index 50%. Which is your
a. s.typhi conclution for this result?
b. shigella dysentriae a. pausibacillar leprosy patient sensitive to
treatment
c. EHEC
b. multibacillar leprosy patient sensitive to
d. s. aureus treatment
e. ETEC
c. multibacillar leprosy patient resistant to A. Reye syndrome
treatment B. Gray syndrome
C. Down syndrome
d. pausibacillar leprosy patient resistant to D. ADHD
treatment E. Black water fever
e. pausibacillar leprosy patient prone to nerve
damage
36. Soalnya ada orang yang mau ke luar dari
jawa dia mau ke papua nah disitu ada banyak
plasmodium tratment apa buat mencegah
33. A boy came with his mother to ER Siloam penyakit tersebut
hospital Jawabannya: doxycycline
His mother said that before this, the boy just
playing in the backyar.
PE: high fever 39'C and multiple stinging on the
back. 37. 40 years old man come to hospital with 4
What is the most likely species that bite him? times generalized tonic clonic seizure since 2
A. Hymenoptera sp. weeks ago. No history of seizure & head trauma.
B. Dermatos andersoni 1 year ago, had bussined in papua for 6 oths.
C. Tactodectus macta During his stay, he ofte osumed undercook pork.
D. Loxoxceles laeta Brain MRI show small cyst??
E. Lytta Vesicatoria
a. taenia eggs

b. cysticercus cellulose
34. A 28 years old lady came to Posyandu to
have a nutrition consultation. Last year, she c. t. solium worm
delivered a stillbirth baby with a spinal disorder.
She and her husband start planning to have a d. t.saginata worm
baby again. She asked for suggestion about her
dietary intake before she got pregnant.
38. 8 day infant born help of ----, irritable 2
What kind of diet does she need to prevent the days ago, noise cause pain episthotonus. PE
same disorder for her future baby? umbilical cord dried. Normal stimulated by
touch/ noise then begin to cry, stiff, arch its
a. Dark green vegetables
back. Stiff continue until calm calm down.
b. Pasteurized milk Diagnosis…
c. Deep water fishes
d. Boiled eggs Bacterial meningitis
e. Lean beef
Botulism

35. 27 yrs old women at term pregnancy had General seizure


fluctuated fever since 2 weeks ago. She looked
pale, generalized weakness and headache.She Tetanus
went to public health and was given Viral encephalitis
chloramphenicol 500mg 4x daily, paracetamol
500mg 3x daily. She is pregnanct and she didn't
tell. What is the side effect drugs will you find in
that baby? 39. A married mom bring her 18 months son to
ER due to rash that develop today. She reports
that he has had a runny nose, conjunctivitis and The baby has cataract a 3/6 systolic heart
diarrhea. On PE he looks ill, temperature 40oC murmur. Enlargement of both liver and spleen,
and diffuse 2-3 mm erythematous rash begin no lymphadenopathy.
from hair line behind ear and spreading to face
and body. On buccal mucosa, red spot are noted. a. Congenital toxoplasma
b. Congenital rubella
What is the most likely causing agent? c. Congenital cytomegalo
d. Congenital herpes simplex
a. Rubella
e. Congenital varicella
b. Coxsackie virus
c. Morbili
d. HSV-6 44. 40 years old woman, presented with
e. Parvovirus B19 raised redness plaque resemble doughnut with
raised border that she already had for 1 year, no
itchy, weakness on her leg. Her slipper left.
.40 : most severe complication from this disease Annular erythematous sharp border, shiny
(morbili) surface, enlarge popliteal nerve. Lab exam?
a. osteomyelitis
b. encephalitis a. KOH
c. otitis media b. Zielh nelson
d. sinusitis c. Gram
e. oral thrush d. Na cl

41. 48 years old acute gastroenteritis has step 45. A girl returned from Lampung with
nadder night fever since 7 days ago. He also has paroxysmal fever, malaise, and myalgia. The
abdominal pain and constipation. Biasa makan fever recurrs every 36 to 48 hour. On
uncooked vege at streets . Which is most likely microscopic examination there are ring and
organism? cresecent like shapes inside the red blood cells.
A. Vi antigen This is characteristic of?
B. Urease A) plasmodium vivax.
C. Hemolysin
D. Shiga toxin B) plasmodium ovale.
E. Pili
C) plasmodium malariae.

D) plasmodium falciparum.
42. Seseorang mengalami sore throat, ada
grayish white di ujung mulut, limfadenopati. E) plasmodium menangle
A. Dipteri
B. Pertusis
C. Tularemia
46. Antidotum untuk keracunan analgesic-
D. ....
antipyretic…
E. ....
a. Amylnitrite
b. Atropine
43. Baby rash identically, mother had febrile c. BAL
illness during the second trimester of pregnancy. d. EDTA
Examination reveals diffuse raised purple skin e. N-methyl cystein
lesion. There is no pallor, jaundice or cyanosis.
47. man 47 years old come to your clinic. he has hypnozoit?
complain about malaise,high fever,and mialgia, A. Primaquine
slightly jaundice.one week ago he has clean a B. Pyrimethamine
canal with his friend..apa yang harus kamu C. Quinine
lakukan sebagai dokter untuk profilaksis dalam D. Chloroquine
kondisi demikian>> E. Chologuanide

a. antiviral,

b. antibiotik 53. Woman 25 y.o, G3 at 39 weeks delivers a


small gestational age infant with chorioretinitis,
c.antitoxin intracranial calcification, micro-cephaly,
jaundice, hepatospleenomegaly. The nurse
d. vaksin reports poor feeding and tone. Mother denies
any raw or undercooked meat, no cats. She
works as nurse in pediatric ICU. What is the
48. women come back from Saudi Arabia, has most likely causative agents?
weakness and tingling sensation in her right leg, A. CMV
other neurology exam is normal including her B. Epstain Barr virus
left leg..Which immunization did she miss? C. Hepatitis B virus
D. Parvovirus
a.MMR E. Yellow-fever virus
b.DPT 54. 45 y.o male has a chronic skin lesion both
hands for 1 year. Skin erythematous, crusted,
e.Oral Sabin
itching, and have serous discharge. Work in a
gold refining plant for 2 years without
protection. Contact with cyanide. Not wearing
49. Seorang anak dengan fever, konjungtivitis, gloves. What is the possible mechanism?
rash dari line belakang rambut kepala sudah A. Immediate reaction
sakit sejak 2-3 hari lalu dari genus apakah itu?? B. Acute reaction
Morbili C. Gradual reaction
D. Delayed reaction
50. Severe complicationnya apa? E. Quantal reaction
Encephalitis
Pilihan yg lain g lupa
55. male, 46 y.o came to you as a doctor with
complaint 'sesak nafas' in
51. 34 yo women, G2 delivers at 36 week of his work place.
growth-restricted infant w/ catarract, PDA, and but when he back to home, 'sesak nafas' is
sensorineural deaf. Mom had flu-like syndrome dissapear. he notice, that
with rash in early pregnancy. Etiology? his sister using broncodilator. he also complaint
A. Porvavirus sneezing in morning.
B. Rubella he work in sandbusting company in last 2 week.
C. Morbili From the following what
D. CMV the appropriate measurement for hazardous
E. HSV thing.
a. Body mass index
52. 27 yo woman, abis travelling southeast asia. b. nutrition
Shaking, chills, 40C. Ada P.vivax. Cr treatment c. activity susceptibility
d. - D. Selalu pake sepatu
e. tissue and organ
E. Jangan digaruk

56. Anak Kecil perempuan berusia 3th


mengeluh tidak mau makan, badannya kecil, A 30 year old woman, presents with abdominal
terdapat anemia, pada pemeriksaan fisik terdapat pain and diarrhea of 3 days duration. She doesn't
hepatomegaly, pada pemeriksaan lab terdapat complaint of nausea, vommiting or fever. She
eosiofilia, kata ibunya dirumahnya terdapat has no sick contact or significant travel history.
anjing dan dia suka main dengan anjing tersebut. A stool sample is obtained, which reveals
rhabditiform larva.
Kemungkinan gejala yang terjadi
59. In condition below adult form of the
a. Creeping Eruption pathogenic agent can be found in the whole GIT
and its larvae can be found in visceral organ
b. Biliary Cirrhosis (lung, liver, gallbladder) is :
c. Cholecystitis a. eosinophilia
b. hyperinfection
d. Pruritus Ani c. retroinfection
d. hypereosinophilia
e. Visceral Larva Migrans e. autoinfection

57. Anak kecil datang ke klinik dengan 60. a woman came with the complaint of being
keluhan gatal2 pada pantatnya, ditemukan uncomfortable and hardened in the left upper
sesuatu seperti kuku, diambil dan didapatkan side abdomen. Every 4 days, she is
telur berdinding radial intermittently feverish, on blood smear exam,
plasmodium parasite (+). Fever symptoms on
Penyebabnya.. previous referred to:
a. Taeniasis Saginata Sporozoit in blood
b. Taeniasis Solium Hypnozoit in liver is found
c. Merozoite discharged from schizont and enter
the blood
d. Oxyuris
Process of gametocyte in blood
e. Ascariasis
Pigment in the parasite is formed

58. Laki2 terkena recurrent fungal infection di


bagian kaki dan gatal2. Dia ke dokter dan telah 61. Symptoms : Fever , nausea, fatigue. Back
dikasih obat. Dia minta advice agar penyakit from Kalimantan. Blood smear revealed
tersebut tdk terulang. Apa yg anda sarankan? schizont isi merozoit 12-24, erythrocyte
enlarges. The pathogen is :
A. Hindari makan seafood
a.VivaxMalaria
B. Selalu pake sandal. b.FalciparumMalaria
c. MalariaeMalaria
C. Jaga tetap kering dan bersih. d. OvaleMalaria
e. DowleskiMalaria is the most effective remains treatement for the
case of pesticide poisoning?
62. 52 years old man is brought to the
emergency department in an unconscious a. Epinephrine
condition on examination, he was death form b. Antacid
more than one hour. On autopsy of the brain, c. Spironoloctate
there is cyst’s fluid containing protoscoceles d. Atropine
with hooklet and many daughter cysts were e. HCT
found. What is the most likely diagnosis of the
disease?
a. Hydatidosis
b. Cysticercosis
c.Serebralmalaria 71. A 65 year old woman suddenly had flaccid
d. Histoplasmosis paralysis at legs. Few days ago, there are
e. Toxoplasmosis headache, nausea, fever, sore throat. Her son:
daughter in law, and young grandchild have
vaccine routine oral.. She taking
imunosuppressant for kidney transplant. Vital
63. dewi…kerjasama nya please…. Demi kalian sign, cranial nerve, CT-scan and MRI normal..
juga kok The transmission?
A. Droplet
64. dewi…kerjasama nya please…. Demi kalian
B. Airborne
juga kok
C.fecal oral
D. Close contact
E.animal bite
65. Pengambilan specimen untuk edema pada
leg dan scrotum pada filariasis adalah terbaik 72. Pork eating village in papua new guinea
pada waktu…. epidemic outbreak of epilecticum seizure. The
investigate:
Answer: 10 – 12 pm A. Level of balantidium coli in swine stool.
B. Practice of consuming raw decease human
brain
C. Presence taenia eggs in drinking water
66. livia…kerjasama nya please…. Demi kalian
D.presence of trophozoite in human blood
juga kok
E. Quantity of culidare in the village
67.livia…kerjasama nya please…. Demi kalian
juga kok
73. 34 years old women complain sudden onset
68. lauren…kerjasama nya please…. Demi
high fever 4 days with nausea, vomite, muscle
kalian juga kok
headache, headache, pretechie + . which of the
69. kesalahan banker… maap yei.. following laboratorium result is the most
appropriate with the patiet diagnosis?

70. 35 years old male, farmer who was working a. trombositopenia


with pestiside was taken to ER with headache,
b. neutropenia
vomiting, salivation, diarrhea, muscle
fasciculation, difficulty walking, difficult c. anemia
speaking. His cloths has been removed, hes been
washed, has been given activated charcoal. What d. hematocrit decrease
e. leukosit decrease e. opiate

74. 1 years old baby get swollen gum 1 week. 77. 22 years old women, G1, complaining flu
On history, he has normal body weight, breast like illness during third trismester early. She
feed until now, balance diet with high intake delivery a growth restricted infant with seizure
cooked vegetable and fruits since 6 month old. intracranial calcification, jaundice. Whats the
No history of cronic disease. Whats the most most possible cause?
likely cause of the complaining??
a. treponema pallidum
a. protein deficiency because of breastfeed
b. T.vaginalis
b. Karbohidrat deficiency because of increase
kalori need c. toxocara sp

c. fat soluble vitamin deficiency because of high d. neisheria meningitis


vegetable and fruit intake
e. toxoplasma gondii
d. water soluble vitamin deficiency because of
cooking

e. mineral deficiency because of competition 78. Influenza virus


with vitamin absorbtion

79. ETEC
75.Male 10 year old,high fever 38 C,runny
nose,sore throat,mild cough, vesicular lesion at
palatum,dorsal,n palmar of the hand. Most likely SOAL UJIAN
diagnosis:
A.adenovirus TROPICAL MEDICINE BLOCK
B.coxsackie virus (ini jwbnnya)
C.
D.sinusitis
E.pneumoni 1. 40 tahun man, fever  1 minggu. Ada
nausea, fatigue. Back from Timika.
Blood smear revealed late trofozoit,
skizon 12 – 24, merozoit 20, RBC
76. 19 years old girl come with sign and enlarges. The cause of the fever:
symptomp of poisoning. Nausea, vomiting, a. Sporozoit in blood
abdominal cramp, diare, excessive saliva, b. Hypnozoit in liver is found
headache, giddiness, rinorhea, pin point pupil, c. Merozoite discharged from
and mental confusion. What ‘s the most possible schizont and enter the blood
agent? d. Process of gametocyte in blood
a. acetaminophen e. Pigment in the parasite is
formed
b. barbiturat 2. 35 years old male, farmer who was
working with pestiside was taken to ER
c. carbamate with headache, vomiting, salivation,
diarrhea, muscle fasciculation, difficulty
d. sianida walking, difficult speaking. His cloths
has been removed, hes been washed, has
been given activated charcoal. What is c. Anti toxin
the most effective remains treatement d. Immunosuppresant
for the case of pesticide poisoning? e. Anti fungal
a. Epinephrine 8. this female illness could be most easily
b. Antacid diagnosed by which of the following
c. Spironoloctate test?
d. Atropine a. HSV – 1 IgM Ab
e. HCT b. CMV antigenemia
3. Man, 31 years old, inguinal c. HHV – 6 IgM Ab
lymphadenopathy, blister on penis, had d. Culture of blood in
unprotected sex 1 month ago. erythroblastic cell
Penyebabnya apa? e. Monospot – test
a. HPV 9. One member of the NGO team from
b. Triponema Pallidum Indonesia has been reported dead in
c. Haemophillus Influenzae Mexico, because of spider’s bite. The
d. Chlamydia trachomatis possible cause of this disorder is:
e. HSV a. Erucism
4. 4 years old, child sleeping scratching b. Lepidopterism
anal. Suspect pin worms. BEST c. Tick Paralysis
diagnosis is? d. Arachnidism
a. Stools for eggs e. Delusional Parasitosis
b. Stools for larva 10. Ulcer + lethargy + pale + face = thin
c. Blood smear of microfilm old person + rib prominent
d. Transparent adhesive tape of a. Marasmus
eggs b. Kwashiorkor
e. Titer IgE antibody c. Marasmic Kwashiorkor
5. Clotted tongue, watery diarrhea, demam d. Energy malnutrition
tinggi pas malem e. Protein malnutrition
a. Salmonella 11. 46 tahun female, several bunion
b. E. Coli removed from her right foot. Chooses
c. Malaria conscious sedation other than general 
d. … become agitated, combative, given IV
e. .. midazolam. Drug that counteract
6. Best treatment for scabies midozolam? (Keracunan obat anesthesia
a. Sulfur midazolam di kasih apa?)
b. Gamexane a. Flumazenil
c. Crotamiton b. Glucagon
d. Lindane c. Naloxone
e. Permethrine d. Nitrite
7. 31 years old female underwent allogenic e. Protamin
bone marrow transplant for AML. 12. 30 years old male, having fever for 3
Second month after, high fever, non days, with falciparum rings +++. He was
productive cough, breathlessness, given CQ3 ( chloroquine for 3 days),
splenomegaly. Lab : anemia, leucopenia, after 3 days, the patients still feverish,
thrombositopenia, abnormal liver blood smear examination show
function test. X-ray: diffuse interstitial falciparum rings ++++. What is the
pulmonary infiltrates. Bronchoalveolar possible respon of CQ?
samnple performed. Infected kidney a. Sensitivitiy
cells  owl eye. Best treatment: b. R1 resistance, early
a. Anti microbial recrudescence
b. Anti viral
c. R1 resistance, delayed d. .
recrudescence e. ..
d. RII resistance 18. female 27 tahun, fever 2 hari,
e. RIII resistance unconscious and general seizure 2 hour
13. Refer to the previous case above, what ago. Physical Examination : GCS 3, BP
is the treatment of choice 90/60 mmHg, temperature 41 C, Hb
a. Primakuin 10.3, WBC 5800, platelet 39.000. blood
b. Amodiakuin smear ada schizont. State yang di takutin
c. Meflokuin setelah ini?
d. Sulfadoxine – pyrimethamine a. …
e. Artemisin b. Intermittent fever
14. 24 tahun female, myalgia and weakness c. Gametocyte formation takes
of her right leg. Dari Saudi Arabia, place in visceral organ
imunisasi ga komplit. Normal sensation d. Make capillary destruction in
on right leg, movement and cranial CNS by parasit contain
nerve normal. Which is a target cell erythrocyte
receptor for this pathogenic agent? e. There are many drugs resist
a. Silica Acid on mucous cell once aginst P.falciparum
b. Ach on neurons 19. A one year old boy was diagnosed with
c. CD 4 on T lymphocyte measles since a week ago. This
d. C2d complement on B morning, his mother noticed that he also
lymphocyte suffered for swollen, reddish gum that
e. Ig on epithelial cell signified as scurvy. Most likely
15. Which one of the following is the best mechanism that underlines his recent
specimen for diagnosing in case above? condition?
a. LCS a. Impaired absorption of
b. Blood micronutrients
c. Urine b. Altered micronutrients
d. Throat swab metabolism
e. Rectal swab c. Depletion of micronutrient
16. Young boy, high fever, difficulty stores
swallowing water, hydrophobia, d. Micronutrient redistribution
disorientation, hypoesthesia. In the left e. Negative mincronutrient
arm, 4 weeks ago was bitten by dog. balance
Characteristic of pathogenic : zoonotic, 20. Group III tropical disease
single stranded, negative sense DNA a. Leprosy, lymphasis filariasis
virus. Which do you found for b. Malaria, TB, Dengue,
diagnosis? Schistosomiasis
a. Giant cell c. Ebola, Avian Influenza, SARS
b. Limfosit plasma biru d. Protein calory malnutrition,
c. Negri bodies pesticide intoxication
d. Inclusion bodies e. Hepatitis B, Hepatitis B,
e. Clue cell pertusis, poliomyelitis
17. Anak laki laki 4 tahun dating dengan 21. Keracunan acetaminophen, kasih apa?
pucat dan intense chills, daily high a. Acetylcysteine
fever. Memiliki sickle cell disease. b. ..
Platelet 53.000, WBC 17.000. c. ..
Description the relevant finding? d. ..
a. .. e. ..
b. .. 22. 45 years old man suffered otitis media 2
c. .. years ago. He has headache a week ago.
Radiographic exam shows 3 cm round b. FVC turun
mass. c. FEV1 turun, FVC turun
a. Neoplasm d. Normal
b. Aneurisma e. MVV naik
c. Infarct 28. Seorang bapak tinggal di Lembah Napu
d. Abscess di Sulawesi, terjadi perbesaran hati,
e. Hemorrhage flatulence
23. Appropriate antiepileptic drug for a. Schistosoma japonicum
Absence Seizure? b. Schistosoma mansoni
a. Carbamazepine c. Schistosoma haemotobium
b. Valproic Acid d. Schistosoma rodentisia
c. Benzodiazepine e. …
d. Lumotrigine 29. Cewe 18 tahun come to obgyn dengan
e. Ethuxusimide no menses 2 bulan, lose weight 5 kg for
24. 52 tahun man is brought to the 3 bulan. Lagi dalam proses pengobatan
emergency department in an TB 4 bulan yang lalu
unconscious condition on examination, a. Increase glukoneogenesis
he was death form more than one houn b. Increase vitamin excretion
on autopsy of the brain, there is cyst’s c. Increase resting metabolism
fluid containing protoscoceles with d. Increase CHO store
hooklet and many daughter cysts were e. Increase protein catabolism
found. What is the most likely diagnosis 30. 70 tahun cowo ke ER gara gara seizure,
of the disease? Istrinya jam 5 pagi bangun, lantaran
a. Hydatidosis suaminya seizure. Kepalanya deviasi ke
b. Cysticercosis kiri, tangannya kenceng, trus ada
c. Serebral malaria generalized body jerking +
d. Histoplasmosis unconsciousness selama 2 menit, stop
e. Toxoplasmosis spontaneously. No history of seizure. He
25. 5 year old girl , 2 hari fever + throat. is drowsy and confused. BP 140/195,
Ada lethargy , grayish white membrane HR 96, RR 20, susuh 36.5 C, Resting
di faring, tonsil asimetris, treatment? Blood Glucose 610.
a. Abx, antiviral a. Pseudoseizure
b. Antiviral, antifungi b. Primary epilepsy
c. Antitoxin, antifungal c. Secondary epilepsy
d. Abx, antitoxin d. Acute provoked seizure
e. Antiviral, antitoxin e. Generalized seizure
26. Shortness of breath at work place. At
home, symptoms missing. Sneeze every
morning. Worked at sandblasting 31. Treatment  Insulin cutaneous
company and expose to dust since 2
weeks ago. Possible cause:
a. Wood 32. Female tourist developed gastroenteritis
b. Quartz and tasted some Indonesian food. Onset
c. Tobacco smoke abrupt with abdominal discomfort and
d. Coal watery diarrhea. Setelah di periksa,
e. Ammonia ditemukan pathogen, yaitu…..
27. doctor gave him sick leave certificate 2 a. Salmonella typhii
days and asks the patient to come back b. Shigella dysentriae
tomorrow to spirometry. Result of c. Enterohemorrhagic coli
spirometry do you expect? d. Enterotoxigenic colli
a. FEV1 turun e. Staphylococcus aureus
33. Seorang pria dari bogor akan pergi ke d. B. paraapertusis
samarinda selama 3 minggu, dikasih e. C. diphtheriae
obat apa? 40. Keracunan parathion 
a. CQ + PQ acteylcholinesterase
b. Sulfadoxine + PQ 41. 2 tahun, febrile, deteriorate, worsening
c. Aretusunate + AQ cough and dyspnea. Child had obvious
d. Kina + Tetracycline respiratory distress with retraction and
e. Doxycycline elevated respi rate. X-ray chest
34. 65 tahun female, tiba tiba lumpuh kaki hiperinfiltration, both lundg field. What
kiri. Beberapa hari lalu, sakit kepala, is the lab?
fever, sore throat, nausea. Tinggal ama a. Nasopharyngeal washing
anak laki lakunya, menantu perempuan, b. Trache aspirate
dan cucu yang masih kecil yang baru c. Bronchial washing
dapet imunisasi rutin, si nenek itu pake d. Nasal swab
immunosuppressant untuk transplant e. Blood
ginjal. Vital bagus, cranial nerves ok. 42. At nite, 28 years old, male, right foot
Head CT normal, Lumbar MRI normal. bitten by animal 1 hour before. Heard
Cara transmisi? the sound of the animal. Pain in Right
a. Droplet foot, 2 bitten mark, edema, erythema, in
b. Airborne dorsal right foot. What likely animal?
c. Fecal-oral 43. Initial management?
d. Close contact a. Obvservation
e. Animal bite b. Cryotherapy
35. 35 tahun male, seizure, CSF = neutrofil, c. Antivenom
RBC, amoeba form on wet microscopy d. Inicision and suction
a. Naegria fowleri e. Corticosteroid
b. Entamoeba histolytica 44. Next step:
c. Acanthamoeba costelorii a. Wide excision
d. Escherichia colli b. Fasciotomy
e. Plasmodium falciparum c. Antivenin
36. 22 tahun, female, 2 week di Papua, d. Incision and suction
blood and slimy diarrhea dari lasat 5 e. Corticosteroid
days in Papua. > 5 kali bowel movement 45. after 12 hours, severe pain especially
each day, stomach cramp, pain stool, no more right leg and leg pallor, cold, no
fever, abdomen tender. Agak dehidrasi pulse in R dorsal artery. Treatement???
a. E.histolytica 46. a woman came with the complaint of
b. Cholera being uncomfortable and hardened in
c. Rotavirus the left upper side abdomen. Every 4
d. Salmonella days, she is intermittently feverish, on
e. E.colli blood smear exam, plasmodium parasite
37. Tidak inget udah imunisasi TT atau (+), of the following, which parasite can
belum, prophylaxis nya apa? cause this?
38. Seizure karena tetanus disebabkan oleh? a. P. falciparum
39. 3 years old boy, brought to emergency b. P.vivax
unit in Tangerang hospital with stridor, c. P.malariae
dyspnea, and croupy cough. What is d. P.ovale
most likely pathogenic agent that cause e. ……
the patient’s disease? 47. Lab test buat pertusis?
a. B. pertusis 48. 10 tahun, diarea, blood and mucus,
b. H. influenzae rectal prolapsus
c. Coxsavirus a. Ascaris
b. Necator j. Nasopharyngeal swab
c. Ancylostoma
d. Trichius Question 3-4 are linked to the following case:
e. Enterobius (RABIES)
49. parasit ini tinggal di? 3. A young man was admitted to hospital after
a. Liver increasing left arm pain and paresthesia.
b. Lung Several days ago, he contacted with a
c. Small bowel dog……. His symptoms increased and were
d. Stomach accompanied by hand spasm and sweating
e. Caecum on the right side of the face and trunk. This
50. Apa yang bisa bunuh virus rabies? patient was admitted to the hospital the day
a. Infrared radiation after developing dysphagia, hydrophobia,
b. Catalase hypersalivation, and disorientation.
c. Alkali water Which of the following is usefull for making
d. Sunlight a diagnosis in this case ?
e. Heating at 60 C for 30 minute f. Giant cell
g. Limfosit plasma biru
h. Negri bodies
CUCU i. Inclusion bodies
j. Clue cell
(HENDRA-NIPAH VIRUS)
1. A 27-year-old man develops acute severe 4. Which of the following is available for
encephalitis that requires hospitalization. treating this patient?
Several days before he got that disease, he a. Immune globulin
recognizes that many pigs in his farm were b. Live attenuated vaccine
died. c. Antiviral
Which of the following is most appropriated d. Antimicrobial
natural host for the pathogen? e. Antitoxin
f. Arthropod
g. Horse
h. Fruit bats
i. Squirrel Question 5-6 are linked to the following case:
j. Rats (ANTRAX-CUTANEOUS)
A 32-year-old worker from a farm comes with a
2. A 32-year-old female developed a “flu-like” painless dermal papule on his right hand which
syndrome with high fever up to 400C, started to develop since five days ago. The
anorexia, headache, and myalgia. Four days worker says as the lesion getting bigger, and the
later, she become confused and agitated with skin becomes black. He has tender axillaries
difficult to breath. Chest x-ray showed lymph node enlargement.
parenchymal infiltrate. One of her kids has
also these symptoms, and has already passed 5. What antibiotic should the doctor give to
away. In order to provide laboratory treat the worker?
conformation of the pathogenic agent, a f. Tetracycline
tissue culture was ordered. g. Aminoglycoside
Which of the following would be the best h. Ciprofloxacin
specimen for isolating the pathogenic agent i. Neomycin
responsible for this infection? j. Crystalline penicillin G
f. Stool
g. Blood 6. Which of the following virulence factors is
h. Saliva most likely to be involved in the
i. Lung biopsi pathogenesis of illness?
f. Exotoxin
g. Endotoxin 10. High priority bioterrorism agents include
h. α-hemolysin organisms that pose a risk to national
i. Lipopolysaccharide security because they can be easily
j. Antiphagotic factors disseminated or transmitted from person to
person. Which of the following is most
(ANTHRAX-GI) likely to be transmitted from person to
7. A 53-year-old male farmer developed low- person in bioterrorism act?
grade fever, abdominal pain, and diarrhea. a. Bacillus anthracis
He has a skin lesion of black eschar b. Clostridium botulinum
surrounded by vesicle and edema near his c. Francisella tularensis
mouth. A week before, his notice many d. Variola major
cattle in his farm were sick, but he still eats e. Brucella melitensis
a half-done beef steak from his cattle.
What is the following microorganism is Question 11-12 are linked to the following case:
most likely involved in this case? (AVIAN FLU)
f. Bacteroides fragillis A 31-year-old man presented with complaints of
g. Leptospira interrogans high fever, cough, and shortness of breath. Two
h. Bacillus anthracis ours before admission, he experienced right-
i. Pasteurella pestis sided chest pain when took a deep breath or
j. Borrelia burgdorferi coughed. Chest film showed diffused bilateral
interstitial pulmonary infiltrate. Arterial blood
Question 8-8 are linked to the following case: gases showed a PO2 of 60 mmHg with 91%
(EMERGING AND RE-EMERGING hemoglobin saturation. The hematologic, serum
INFECTIOUS DISEASE) and liver tests were normal. He lives near the
There is an RNA virus that causes deadly poultry, and the pathogenic agent is a new virus.
outbreaks of hemorrhagic disease in Africa. In
each outbreak, hospital staff became infected. 11. Which of the following is the prevention for
This virus is highly virulent. this disease?
f. Vaccine
8. Which of the following is the most likely g. Antitoxin
transmission for this disease? h. Antibiotic
f. Spread by contact with blood or other i. Antiviral
body fluid j. Antidotes
g. Transmitted by mosquitoes bite
h. Transmitted to human from rodent 12. What is the following is probably the reason
excreta for the emergence of this infectious disease?
i. Aerosol transmission a. Animal husbandry practice
j. Direct contact with cattle product b. Genetically engineering
c. Vector elimination
d. Antibiotic resistance
e. Tsunami impact

13. An 80-year-old man is developing fever,


9. Which virus is most likely the cause of this cough, runny nose, headache, and myalgia.
outbreak? He lives in a nursing home. Many of people
a. Dengue virus in his neighborhood have this disease.
b. Menangle virus Which immediate course of action would be
c. Lassa virus most appropriate for this patient?
d. Ebola virus a. Vaccination
e. Influenza virus b. Immunoglobulin
c. Rimantadine vaccinations. His vital signs revealed
d. Acyclovir bradycardia and fever of 400C. His physical
e. Cytokine examination revealed mild hepatospleenomegaly
and faint erythematous macules.
14. A 22-year-old man who works in the
pediatric ward of a hospital suffers from 17. Which of the following is most likely to
malaise, sneezing, and runny nose. He have caused this man’s illness?
subsequently develops a mild sore throat, f. Vibrio cholerae
headache, and stuffy nose. The symptoms g. Shigella dysentriae
resolve within 4 days. Which virus is most h. Salmonella typhi
likely to be responsible for these symptoms? i. Entamoeba histolitica
f. Rota virus j. Giardia lambia
g. Rubella virus
h. Coxsackie virus
i. Hepadnavirus
j. Influenza virus

(TIFOID)
15. A 48-year-old man with acute gastroenteritis 18. Which of the following specimen is most
has sub febrile fever since 7 days ago. He appropriate for case above?
also has abdominal pain, and diarrhea. He f. Feces
usually eats undercooked egg every g. Blood
morning. h. Urine
Which of the following is most likely to be a i. Throat swab
constituent of this organism? j. Biopsy
f. Vi antigen
g. Urease Question 18-20 are linked to the following case:
h. Hemolysin (MALARIA)
i. Shiga toxin A 24-year-old man returned from Papua Island.
j. Pili Five days later, he developed repeating intense
chills and high fevers. These severe episodes of
16. An outbreak investigation note many fever had been occurring every other day. In
costumer from a café were admitted to the between these episodes, he had low-grade fever,
hospital. They have fever; nausea-vomiting, myalgia, nausea, vomiting, and diarrhea. A few
constipated or diarrhea, weakness and hours ago, he was admitted to ICU in coma. He
altered mental status. Rose spots are seen on became progressively somnolent and died a
the trunk. Blood cultures from patients grow week later.
a non-lactose-fermenting gram-negative rod.
In which of the following sites is a 20. Which of the following organs does this
bacterium most likely to be found? infectious agent initially proliferate after
f. Blood entry in the infected host?
g. Kidney f. Heart
h. Liver g. Liver
i. Intestine h. Brain
j. Gall bladder i. Spleen
j. Renal
Question 17-18 are linked to the following case:
21. What is the most common mode of
A 28-year old man presented with 6-day history transmission of this agent?
of increasing fever, malaise, headache, and a. Contact with rat’s of urin
constipation. He did not receive any prior b. Transplacental crossing
c. Blood transfusing investigate. One of the first things you
d. Bite of a mosquito should investigate is?
e. Eat the contaminated food f. The level of Balantidium coli in swine
stool
22. An apparently run-down but alert 34-year- g. The practice of consuming raw human
old woman comes to your office after 6 brains from recently deceased family
months spent as a teacher in a rural West members
Sumba. Her chief complaints are frequent h. The presence of taenia eggs in the
headaches, occasionally nausea and drinking water
vomiting, and periodic fever. To rule out i. The presence of tropozoites in the
your differential diagnosis, a smear of human blood
finger-stick blood was done. Which of the j. The quantity of culidae in the village
following choices would fit your diagnosis
based on your microscopic examination of
the blood smear? dr. ANGELA
f. Schizonts in red cells with 8-12 progeny
g. Rounded gametocytes present Question 25-27 are linked to the following case:
h. Enlarged, somewhat misshapen (BRUCELLOSIS)
parasitized red cells A 45-year-old man from the Middle East had
i. Numerous large ovoid parasites in some fever and chills, with weight loss, sweats,
of the red cells headache, muscle pain, fatigue, and depression.
j. Numerous band-shaped tropozoites in From physical examination the doctor found
the parasitized red cells lymphadenopathy and spleenomegaly. The man
is a daily farmer and a couple weeks before the
symptoms appeared he drank a glass of
unpasteurized cow milk. The culture of blood
grew a tiny Gram-negative coccobacilus,
catalase and oxidase-positive; that resemble fine
grains of sand.

25. The patient was probably infected with


(NEUROCYSTICERCOSIS) which of the following microorganism?
23. A 39-year-old woman live in Manokwari, f. Mycobcaterium tuberculosis
Papua Island developed a major seizure g. Salmonella typhi
while at work. She had no history of h. Brucella species
epileptic disease. A head MRI was i. Pasteurella pestis
remarkable for a lesion surrounding a j. Staphylococcus aureus
scolex. The etiologic agent would most
plausibly have been acquired by eating or 26. Which of the following diagnostic
dinking which of the following food items? laboratory test should be done for
f. Uncooked vegetables identification of this fastidious
g. Raw oysters coccobacilus?
h. Raw pork a. Fluorescent antibody test
i. Uncooked fish b. Western Blot
j. Unfiltered water c. Culture
d. ELIZA assay
24. A pork-eating village in the highlands of e. PCR
Papua New Guinea is reported to be
suffering from an epidemic outbreak of
epileptiform seizures. You have been sent to 27. How did the man get infected form this
disease?
a. Ingestion of infected unpasteurized cow g. Pertusis
milk h. Weil’s disease
b. Needle stick injury from a contaminated i. Lyme disease
syringe j. Relapsing fever
c. Direct contact with infected animal parts
through intact skin
d. Aedes aegypty bites
e. Fresh orange juice consumption

Question 28-29 are linked to the following case: 31. Which of the following disease is the
(TULAREMIA) complication of this disease?
A 55-year-old warden found a dead muskrat on f. Pulmonary hemorrhage
the bank of a stream. He picks up the animal and g. Myopericarditis
buried it. Four days later, he developed a 1.5 cm h. Aseptic meningitis
painful ulcer on the index finger of his right arm, i. Osteomyelitis
a 1 cm ulcer on his right forehead, and pain in j. Anemia
his right axilla. Physical examination also
revealed right axillary lymphadenopathy. 32. Which of the following reservoir animal is
the source of human infection?
28. Which of the following is working diagnosis a. Fish
for this patient? b. Bird
f. Brucellosis c. Reptile
g. Pertusis d. Scallop
h. Leptospirosis e. Rat
i. Tularemia
j. Hydatidosis 33. Which of the following is drug of choice of
this disease?
29. Which of the following prevention should be a. Penicillin G
done for high risk persons, such as research b. Chloramphenicol
laboratory personnel from this disease? c. Erythromycin
a. Education d. Gentamycin
b. Immunization e. Rifampin
c. Laboratory precaution
d. Burning infected animal Question 34-35 are linked to the following case:
e. Antibiotics taken (LEPTOSPIROSIS)
A 27-year-old medical student was admitted to
Question 30-33 are linked to the following case: hospital because of sudden onset fever up to
(LEPTOSPIROSIS) 390C and headache. Two weeks previously he
A volunteer man returning from a tsunami area volunteers cleaning the canal with others. Blood
was admitted to hospital. A week before, he had tests done shortly after admission indicated renal
fever and headache. These symptoms resolved, function abnormality and elevated liver function
but the day before admission he became pyrexial tests. On examination, he looked so yellow.
and on examination was found to be jaundice
and to have an elevated blood urea. Urine was 34. Which of the following would be most
collected and inoculated into a semisolid agar likely to confirm the diagnosis?
medium, and examined by dark ground a. Testing serum using the rapid plasma
microscopy. reagin (RPR) test
b. Culture the urine on human diploid
30. Which of the following is your working fibroblast cells
diagnosis? c. Testing serum by darkfield microscopy
f. Syphillis
d. Testing serum for antileptospiral
antibody dr. ROBERT TEDJA
e. Culture of CFS on chocolate agar
Question 39-40 are linked to the following case:
35. Which of the following is the prevention (TRAVELER DIARRHEA AND THE
after exposure? PREVENTION)
f. Human vaccine A female tourist developed gastroenteritis while
g. Health education visiting small town in Indonesia and tried some
h. Vector controll Indonesian traditional food. The onset of the
i. Isolation the infected people disease is abrupt with abdominal cramps and
j. Antibiotics prophylaxis watery diarrhea. She had no fever or nausea or
vomiting. The symptoms have resolved within
24 hour and no subsequent recurrences. They
report the disease to district public health office.
The investigation found that one of the food
products eaten by this tourist was contaminated
by suspected pathogens.
Question 36-38 are linked to the following case:
(YAWS) 39. What is the suspected pathogen may cause
A young boy had an ulcerating papule on his the disease above?
legs. Later, this lesion destroys the bone of his k. Salmonella typhi
legs (gummata), but three are no visceral and l. Shigella dysenteriae
nervous system complications. The disease is m. Enterohemorragic E. coli
still endemic in hot tropical countries, such as n. Staphylococcus aureus
Indonesia. o. Enterotoxigenic E.coli

36. Which of the following microorganism is


the cause of the disease above?
a. Treponema pertenue 40. Which of the following is the natural
b. Borrelia burgdorferi prevention for the disease above?
c. Leptospira interrogans a. Bi Subsalicylate
d. Spirillum minor b. Antibiotic
e. Chlamydia trachomatis c. Probiotic
d. Chlorination
37. Which of the following drugs is the drug of e. Hand hygiene
choice for the disease above?
a. Tetracycline 41. Approximately 4 hour after eating a meal in
b. Penicillin restaurant, 3 members of a tourisms group
c. Doxyccycline develop a sudden onset of nausea, vomiting,
d. Vancomycin severe abdominal cramps, and diarrhea.
e. Clarithromycin Nobody got febrile. Which of the following
vaccination is should be given to the
38. Which of the following examination is the traveler?
most appropriate to diagnosis the disease f. Shigella vaccine
above? g. Salmonella vaccine
f. Widal test h. Enterobacter vaccine
g. IgG and IgM of Treponema i. H Pylori vaccine
h. Dark-field microscope j. Cholera and ETEC vaccine
i. FTA-ABS test and TP-PA test
j. ELIZA assay dr. SANDRA
(DHF)
44. Which of the following examination is the
42. A 23-year-old man sees his family physician most appropriate to diagnose this infection?
with a sudden onset of 4-day history of f. Prick test
fevers, headache, retro-orbital pain, myalgia g. Tourniquet test
and rash. Physical examination shows h. Complete blood test
diffuse erythroderma with blanching i. Widal test
erythema and petechial formation resulting j. Urine test
from pressure applied to her skin.
Laboratory test reveal leucopenia and 45. The patient was give IV line, and blood
trhrombocytopenia. Which of the following examination was done. Which of the
virus is most likely responsible for this following result is most appropriate with the
infection? patient’s diagnosis?
f. Morbili f. Trombositosis
g. Dengue g. Leukocytosis
h. Influenza h. Erythrositosis
i. Coxsackie i. Elevated of hematokrit
j. Rhinovirus j. Elevated of LED

43. A 34-year-old woman complained a sudden 46. In the forth day, the patient developed
onset of high fever for 3 days with nausea, epistaksis, melena and more clear
vomiting, headache, muscle ache. There was hemorrhage in skin. Which of the following
petechie on examination. The laboratory test is responsible for his condition?
revealed leucopenia, thrombocytopenia, and a. Elevated of thrombocyte increasing
elevated of hematocrit. The patient most coagulation
likely acquired this infection by which of the b. Fibrin degradation make dysfunction of
following modes? thrombocyte
a. Intravenous drug use c. Elevated of kinin in circulation
b. Inhalation of contaminated dust
c. Close contact with a symptomatic d. Plasmin elevated coagulation factor
patient e. Complemen activation by plasmin
d. Bitten by an arthropods
e. Eating some contaminated food dr. DWI

47. A 48 year old lady came to the outpatient


clinic with chief complaint of fatigue and
difficulty in concentration for the last 2
weeks. The history taking revealed that she
had a balanced diet, no history of chronic
disease and had been a heavy smoker for 20
years. On physical exam it was found that
Question 44-46 are linked to the following case: she had anaemic conjunctiva with other
findings was within normal limit. The
A 20-year-old woman came to the emergency laboratory examination showed that she had
room with high fever from 3 days ago. She also haemolytic anaemia.
has headache and pain in retroorbita, abdominal Based on the information given above, what
pain and bloody stool. On physical examination, is the most likely cause for her condition?
she was in an agitated condition. Her BP 90/60 f. Vitamin K deficiency
mmHg, heart rate 110x/ minute, RR 20x/ minute g. Copper deficiency
and there was red spot in her arm, hand and leg. h. Vitamin E deficiency
i. Iron deficiency
j. Folate deficiency
lung, dullness on percussion. Laboratory result
indicated leucopenia.
48. A 28 year old lady came to the Posyandu to
have a nutrition consultation. Last year, she 50. Which criteria of avian influenza is the most
delivered a stillbirth baby with a spinal appropriate with this patient’s condition?
disorder. She and her husband start planning a. Probable
to have a baby again. She asked for b. Suspect
suggestion about her dietary intake before c. Confirmed
she got pregnant. d. Definitive
What kind of diet does she need to prevent e. Absolute
the same disorder for her future baby?
f. Dark green leafy vegetables
g. Pasteurized milk
h. Deep water fishes
i. Boiled eggs
j. Lean beef

49. A 1 year old girl came to the Puskesmas 51. Based on its ability to cause disease, avian
with swollen gum for a week. From the influenza is broadly divided into highly
history taking it was found that she was born pathogenic (HPAI) and low pathogenic
normal with normal birth weight, had been (LPAI) strains. Which subtypes are known
breastfed up to now and had a balanced diet to cause the HPAI?
with high intake of cooked vegetable and a. H1 and H3
fruit since 6 months old. She had no history b. H2 and H4
of chronic diseases. Her other physical and c. H5 and H7
laboratory examination was normal. d. H6 and H8
Based on the information given above, what e. H9 and H11
is the most likely cause of her complaint?
f. Protein deficiency due to prolonged 52. A 50 year old woman presented to clinic
breastfeeding with fluctuated fever (also called as step
g. Carbohydrate deficiency due to ladder fever) since 7 days ago. On physical
increased calorie need examination, you found “rose spots” on
h. Fat soluble vitamin deficiency due to upper abdomen. What is the possible cause
high vegetable/fruit intake of rose spots on this woman?
i. Water soluble vitamin deficiency due to a. Eritrocytopenia
cooking process b. Leucopenia
j. Mineral deficiency due to competition c. Thrombocytopenia
with high vitamin absorption d. Limfositosis
e. Deficiency of clotting factor
dr. JULIANA
(TUTOR) 53. A 27 year old woman in pregnancy had
fluctuated fever since 2 weeks ago. She
Question 50-51 are linked to the following case: looked pale, generalized weakness, and
headache. She went to public health center
A 9 year old girl came to the emergency room and was prescribed oral chloramphenicol in
with difficult of breathing. She had got high doses of 500 mg four times daily and
fever since 3 days ago. Her temperature was up paracetamol 500 mg three times daily. She
to 40oC. She also complained of rhinnorhea, did not tell to the physician that she was
cough, and myalgia. On examination, you found pregnant. What is the side effect of
increased tactile fremitus on the right side of the chloramphenicol on this patient?
f. Reye syndrome
g. Gray syndrome generalised tonic seizure. Her clinical
h. Down syndrome examination was normal. Within a few
i. ADHD hours of her admission she was found dead
j. Black water fever in her bed during the ward round. After
autopsy, the diagnosis was
54. A 24 year old man, recently returned from neurocycticercosis. Which of this following
Myanmar, presented very unwell febrile. His brain’s location is the possible cause of
consciousness was GCS of 7. His thin blood patient’s sudden death?
smear showed malaria falciparum. Which of a. The frontal lobe
this following is the best treatment for this b. The temporal lobe
patient? c. The parietal lobe
a. Chloroquine d. The lateral ventricle
b. Mefloquine e. The fourth ventricle
c. Fansidar
d. Doxycycline dr. ROBERT SOETANDIO
e. Quinine IV
58. You are precepting a resident who has just
evaluated a 4-year-old incompletely
immunized immigrant boy who has classic
varicella lesions and a history that is
55. A 35 year old man, a businessman, came to consistent with this diagnosis. Of the
private doctor. He asked the physician about following, the MOST accurate statement is
his plan to go to Africa, which is endemic that
malaria. The physician suggested him to a. Lesions of both varicella and smallpox
take prophylactic drug. Which medication follow a 7- to 10-day course from
(WHO recommended) would be given to eruption to resolution
this patient for prophylactic? b. Lesions of both varicella and smallpox
f. Kina frequently produce deep, pitted scars
g. Chloroquine c. Varicella lesions appear in stages or
h. Doxycycline crops; smallpox lesions are uniformly in
i. Kuinin the same Stage of development
j. Primakuin d. Varicella lesions are concentrated on the
face; smallpox lesions are concentrated
56. A 40 year old man came to hospital with over bony prominences
four times generalized tonic-clonic seizure e. Varicella lesions are transient vesicles;
since two week ago. There was no history of smallpox lesions are persistent pustules
seizure and head trauma. A one year ago, he until resolution of the illness
had business in Papua for six months. 59. You are working in a refugee camp when a
During his stay in there, he often consumed mother brings in her 8-day-old boy. The
under-cooked pork. Brain MRI showed a mother states he started becoming irritable 2
small cystic with invaginated scolex inside. days ago, and now any loud noise appears to
What of this following is the most likely cause him pain, as evidenced by muscle
inside cyst? tightening and back arching causing his head
f. Eggs to nearly touch his feet. Physical
g. Cycticercosis cellulose examination reveals only a dried packing on
h. Cysticercosis bovis his umbilical cord, as is the local custom. He
i. Taenia solium appears normal until he is stimulated by
j. Taenia saginata touch or a loud noise, and then he begins to
cry, stiffens, and arches his back. The
57. A 15 year old girl of African origin was stiffness continues until he calms down. Of
admitted with a history of headaches and a the following, the MOST likely diagnosis is
f. Bacterial meningitis
g. Botulism
h. Generalized seizure 63. What is the name disease what is caused by
i. Tetanus coxsackie virus and the location in the
j. Viral encephalitis mouth?
a. HFMD
Question 60-61 are linked to the following case: b. Herpangina
c. Conjungtivitis
60. 4. A worried mother brings her 18-month- d. Pleurodynia
old son to the emergency department e. Streptococcal pharyngitis
because of a rash that developed today. She
reports that he has had a runny nose, Question 64-65 are linked to the following case:
conjunctivitis, and diarrhea. On physical
examination, he appears toxic, is 64. A 10-year-old girl presents with a history of
temperature to 104ºF (40°C) for the last 3 sore throat and difficulty breathing of 1
days and has diffuse 2- to 3-mm day’s duration. She returned 2 days ago
erythematous rush beginning from their from a trip to Odessa (Ukraine). On
face. Of the following, the MOST likely examination she is very ill-appearing. She
cause of the rash is has some inspiratory stridor and thick white-
f. Adenovirus gray material covering her tonsils and
g. Coxsackievirus faucial pillars, and she has swelling of her
h. Measles neck, no splenomegaly. The lymphocyte is
i. Human herpesvirus 6 normal. What is the likely diagnosis?
j. Parvovirus B19 f. Tonsilitis Folikularis
g. Angina Plaut Vincent
61. Case no 4, if the son got worse. What would h. Mononucleosis infectiosa
the complication happened? i. Laryngitis Akut
f. Osteomylitis j. Faucial Diphteria
g. Encephalitis
h. Otitis externa 65. Case no 64. if we want to give her DAT.
i. Sinusitis How much should we give DAT?
j. Oral thrush a. DAT 80.000 IU
b. DAT 50.000 IU
Question 62-63 are linked to the following case: c. DAT 100.000 IU
d. DAT 40.000 IU
62. A 10-year-old boy who has moved to your e. DAT 60.000 IU
practice recently has sore throats, get high
temperature. The child reports a runny nose, 66. According IDAI schedule 2008, when is the
mild cough, and abdominal pain. Findings child getting the vaccination booster for the
on physical examination include a disease above after 18 months?
temperature of 100.3°F (38°C), and a. 5 years
vesicular lesions on the soft palate. There is b. 4 years
no cervical adenopathy or rash. c. 3 years
Of the following, the MOST likely diagnosis d. 12 years
is e. 15 years
f. Adenovirus infection
g. Coxsackie virus infection Question 67-68 are linked to the following case:
h. Mononucleosis
i. Sinusitis 67. A 16-year-old boy presents with an
j. Streptococcal pharyngitis annoying cough, which he has had for about
3 weeks. The illness started with a runny
nose. On examination he is completely 71. How old is the baby getting the vaccination?
normal, but he exhibits several episodes of a. 15 months
severe coughing. He is difficult to get b. 16 months
inspiratory and usually finished by vomit. c. 17 months
The laboratory results are leucocytocis and d. 18 months
absolute lymphocytes. Of the following, the e. 19 months
MOST likely diagnosis is
f. Tonsilitis Folikularis
g. Bronchiolitis dr. HANNA
h. Pertussis Question 72-73 are linked to the following case:
i. Laryngitis Akut
j. Faucial Diphteria A 45-year-old man was seen in the dermatology
clinic because of some nodules on his arm and
body and sometimes feel pain with pressure. On
examinations we found erythematous nodules,
68. The boy in case 67 often gets secondary smooth and shiny, diffuse infiltrate with ill
infection. What the most organisms is define border, no fluctuation or erosion. There
usually cause the secondary infection? were pain on his elbow with nerve
a. Stafilokokus aureus enlaergement.
b. Treponema palidum
c. Haemophilus Influenza 72. What is the most likely diagnosis?
d. Serratia marcescens a. Yaws
e. Corynebacterium diphteriae b. Leprosy
c. Anthrax
Question 69-71 are linked to the following case: d. Cellulitis
e. Carbuncle
69. A baby is born with a rash identical.
Questioning reveals that the mother had a 73. What laboratory test would you do to
febrile illness during the second trimester of confirm the clinical diagnosis?
pregnancy. Examination reveals diffuse f. TPHA and VDRL
raised purple skin lesions. There is no pallor, g. Gram preparation
jaundice, or cyanosis. The eyes are cataract, h. KOH preparation
the heart has a 3/6 systolic murmur, and
there is enlargement of both the liver and the i. Skin scrapping
spleen. There is no lymphadenopathy. j. Skin slit smear
Of the following, the MOST likely diagnosis
is? 74. The 21-year-old man was seen in
f. Congenital Toxoplasmosis dermatology clinic because of sore and
g. Congenital Rubella redness on his previous skin lesion. He was
h. Congenital Cytomegalovirus on M D T treatment since 1.5 year ago. On
i. Congenital Malaria skin examination they found some painfull
j. Congenital Varicella erythematous nodule spread over his body
especially on arm and legs also fever. What
70. What is vaccination which prevents disease is the most likely diagnosis ?
in number 69? a. Celulitis
a. Measles b. Furuncle
b. Varicella c. Erythema nodosum
c. MMR d. Multibacillar leprosy
d. HPV
e. PCV e. Pausibacillar leprosy
75. A laboratory examination of a leprosy 78. A patient on MDT treatment complaining
patient after 4 months M D T therapy about red urine that he experinced on the
revealed Bacterial index 6+ and first day of treatment. What is the suggestion
morphological index 50%. What is your for this condition ?
conclution for this result ? a. Stop Clofazimin, can cause pink
f. Pausibacillar leprosy patient sensitive to brownish discolouration
treatment b. Continue the medication, its not a drug
g. Multibacillar leprosy patient sensitive to adverse effect
treatment c. Stop the medication because of
h. Multibacillar leprosy patient resistent to nefrotoxicity
treatment d. Stop Dapsone can cause anemia
i. Pausibacillar leprosy patient resistent to hemolytic
treatment e. Change the treatment to single drug
j. Pausibacillar leprosy patient prone to therapy
nerve damage
79. A 50-year-old man complained about the
76. A 40-jear-old woman presented with a treatment for his leprosy since it takes one
raised redness plaque resemble a doughnut year medication with multiple drugs (MDT)
with a raised border that she already had for every day. Can you give some explaination
1 year, It was no itchy, no pain, not easily about the length of treatment ?
bleeding. Sometime she feel weakness on a. There is no cure for leprosy
her leg and her sandal left behind while b. Leprosy bacteria can’t be killed by 1
walking. On her legs she developed an drug
anular erythematous sharp border lesion, no c. M D T treatment can cure the disability
scally, shiny surface and enlargement of the
popliteal nerve. What is the most likely d. To prevent from drug resistancy
diagnosis? e. Due to his low immunity
a. Leprosy
b. Yaws dr. KRISTO
c. Celulitis
d. Anthrax 80. Tuberculosis become one of reemerging
infectious disease because
e. Insect bite a. More cases of tuberculosis found in
developing countries
77. A 30-year-old farmer complaned about b. Found more cases in childrek
weakness on his three fingers that he can not c. Found more cases of lung cancer
hold a plough He found some white patches d. Found more cases of drug resistance
on his back, tummy arms, he felt numbness e. More people who smoke
on his right hand but not itching After one
year on medication he experience tenderness
and sore on his right elbow and the skin 81. Antigenic shift of influenza viruses occurred
patches became red. What is the most likely in
diagnosis? a. Haemagglutinin
a. Framboesia b. Neuramidase
b. Scorpion bites c. M2 protein
d. NS1
c. Reversal reaction e. NS2
d. Lepromatous leprosy
e. Ramsay Hunt syndrome 82. Epilepsy which occurs in neurosistiserkosis
caused by the
a. Colloidal cyst
b. Vesicular cyst patients with typhoid fever was the doctor
c. Calcification cyst taking the material for culture from?
d. Necrosis cyst a. blood
e. Granuloma cyst b. urine
c. stool
83. Human infection with the beef tapeworm, d. intestinal secretion
Taenia saginata, usually is less serious than e. bone marrow
infection with the pork tapeworm, Taenia
solium, because 87. Typhoid fever pathogenesis is not affected
a. Acute intestinal stoppage is less common by
in beef tapeworm infection a. Antigen Vi
b. Larval invasion does not occur in beef b. Antigen H
tapeworm infection c. Gastric acidity
c. Toxic by-products are not given off by the d. Amount of bacteria
adult beef tapeworm e. Antacid drug
d. The adult beef tapeworms are smaller
e. Beef tapeworm eggs cause less irritation 88. Amantadine, a synthetic antiviral agent used
of the mucosa of the digestive tract prophylactically againt influenza, is thought
to act by
84. A woman, recently returned from Lampung, a. preventing production of viral capsid
complains of having paroxysmal attacks of protein
chills, fever, and sweating; these attacks last b. preventing virion release
a day or two at a time and recur every 36 to c. preventing penetration of the virus into
48 h. Examination of a stained blood the host cell
specimen reveals ringlike and crescent-like d. preventing uncoating of viral DNA
forms within red blood cells. The infecting e. causing lysis of infected host cells by
organism most likely is release of intracellular lysosomal
f. Plasmodium falciparum enzymes
g. Plasmodium vivax
h. Plasmodium ovale 89. A 27-year-old female has just returned from
i. Plasmodium malarie a trip to Southeast Asia. In the past 24 hours,
j. Plasmodium brasilianumi she has developed shaking, chills, and a
temperature 40o C. A blood smear reveals
85. Malaria is a significant worldwide public Plasmodium vivax. Which of the following
health problem. Which one of the following agents should be used to eradicate the
control methods for malaria is currently extraerythrocytic phase of the organism?
effective? f. Primaquine
a. A vaccine g. Pyrimethamine
b. Chemoprophylaxis h. Quinacrine
c. Antibiotics i. Chloroquine
d. White clothing j. Chologuanide
e. Tick repellents
90. The mechanism of action of chloroquine in
86. A man came to the emergency room with Plasmodium falciparum malaria is
complaints body felt weak, fever, and elimination of?
diarrhea since 7 days ago. Prior to the a. Secondary tissue schizons
emergency room, the patient had been to the b. Exoerythrocytic schizoons
clinic 24 hours and given paracetamol and c. Erythrocytic stage
cloramfenicol for 5 days, but the symptoms d. Asexual forms
did not subside. For definitive diagnosis in e. Sporozoites
91. A 26-year-old nulliparous woman, in her
seventh month of pregnancy complains of a 94. A 27-year-old man came to a clinic with
7 days history of fever especially at night. high fever for two days which is started with
As she recall, 2 weeks ago she ate gado- a chill. He also complained of swellings in
gado in coastal area when she out for duty his left inguinal area. They were firm,
for several days. She also had diarrhea and discrete, and somewhat tender. His scrotums
abdominal discomfort. On physical are also enlarged and tender. His doctor
examination reveals blood pressure 110/70 performed a thick blood smear examination
mmHg, heart rate 60 ×/min, RR 20x/min, using blood that was taken at night but
temp. 39.1°C. Tongue is coated and tremor. found nothing. What should the doctor do to
Fetal USG shows normal. make the diagnosis?
What is the most likely etiologic factor a. Biopsy of the scrotum
above? b. USG of the scrotum
f. Salmonella typhi c. CT scan of the scrotum
g. Plasmodium falciparum d. MRI of the scrotum
h. Vibrio cholera e. Aspiration of the scrotum fluid
i. Hepatitis A
j. Escherichia coli
95. A young man presented to his general
92. A young man was admitted to hospital after practitioner with a history of headache and
increasing left arm pain and paresthesia. fever together with a dry cough and muscle
Several days ago, he contacted with a ache. His nose was also blocked and his
dog……. His symptoms increased and were eyes watery.
accompanied by hand spasm and sweating What is your clinical diagnosis?
on the right side of the face and trunk. This a. Influenza virus infection
patient was admitted to the hospital the day b. Human herpes virus
after developing dysphagia, hypersalivation, c. Cytomegalovirus
agitation, and generalized muscle twitching. d. Epstein-Barr virus
Which of the following could rapidly e. Adenovirus
destroy the pathogenic agent of this disease?
a. Infrared radiation 96. A 4-year-old child presents with fever,
b. Catalase cough, conjunctivitis, coryza, photophobia,
c. Alkali water posterior cervical adenopathy. Red lesions
d. Sunlight with a white center are present on the buccal
e. Heating at 60ºC for 30 minute mucosa. A generalized erythematous rash is
also noted. What is the appropriate
93. A 35-year-old man as a farm worker who management of this patient above?
was working with pesticides is brought to a. Immune globuline
the emergency room with headache, b. MMR vaccine administration
vomiting, salivation, diarrhea, muscle c. Measurement of IgG antibody serum
fasciculation, difficulty walking, and d. Measurement of IgM antibody serum
difficulty speaking. His clothing has been e. Antibiotics
removed, he has been washed, and he has
been given activated charcoal. What is the 97. To know the endemicity of malaria on Pulau
most effective remaining treatment for this Seribu, spleen examination on children aged
case of pesticide poisoning? 2-5 years was done. The result shows that
f. Epinephrine this place is mesoendemicity.
g. Antacid What is the meaning of mesoendemicity?
h. Spironolactone a. Spleen rates not exceeding 10 %
i. Atropine b. Spleen rates between 11% and 50%
j. Hidrochlorothiazide c. Spleen rates constantly over 50%
d. Spleen rates constantly over 75%
e. Parasite rate not exceeding10% 102. A patient complains of having nail size
excrement from his anus. On the stool
98. The cattle at animal husbandry are examination, parasite eggs contains a hexacanth
occasionally was dead because of tick embryo with six hooklets surrounded by radially
paralysis, the species of arthropod that can striated spherical shell, 30 to 40 u in diameter
be the cause to this disorder is; were found, the most likely diagnosis of this
a. Rhipicephalus sanguineus patient is :
b. Dermacentos andersoni a. Saginata taeniasis
c. Lactodectus mactan b. Solium taeniasis
d. Loxoxceles laeta c. Cysticercosis
e. Lytta vesicatoria d. Sparganosis
e. Dipylidiasis
99. After contacting with hairy caterpillars, a
farmer felt itching all over the whole Question 92-93 are linked to the following case:
body and his eyes are swollen. The possible
cause of this disorder is: A 40-year-old man came to the clinic with
a. Erucism complaints of having fever more than one-week,
b. Lepidopterism nausea, and fatigue. He came back from Timika,
c. Tick paralysis Papua. On blood smear examination, there are late
d. Arachnidism trophozoites forms, schizonts with 12-24
e. Delusional parasitosis merozoites, and the infected red cell is enlarged.

103. Of the following, the most likely caused


of parasite species is :
f. Plasmodium vivax
g. Plasmodium falciparum
Question 89-90 are linked to the following case: h. Plasmodium malariae
i. Plasmodium ovale
A 23-year-old man presents with extreme j. Plasmodium cynomolgi
swelling of his legs and scrotum. The skin
associated with the swollen areas is thick and 104. Fever symptom on the previous question
scaly. The patient admits to an episode of fever referred to :
associated with enlarged inguinal lymph nodes f. Sporozoites is in the blood
some time ago, but did not think much of it. g. Hypnozoites in the liver is formed
h. Merozoites discharged from schizonts
100. Elephantiasis is caused by the enter the blood
obstruction of : i. The process of gametocytes in the blood
a. The arteries by microfilariae j. Pigment in the parasite is formed
b. The arteries by adult worm
c. The lymphatics by microfilariae 105. A 27 year old women suffering fever for 2
d. The chronic lymphatics by adult worm day. She brought to emergency department
e. Fibrosis of the lymphatic unconsciously and she had general seizure 2
hour ago. On physical examination she is
101. Occult filariasis is Lymphatic filariasis comatous, GCS (Glasgow Coma Scale) : 3;
based upon : BP 90/60 mmHg; t 41 °C. Lab. finding: Hb
a. Infection 10, 3 gr%; WBC 5800/mm3; platelet
b. Allergy 39.000/mm3; there is schizon form on blood
c. Infestation smear. The most likely diagnosis is :
d. Colonization a. Malaria viviax
e. Genetic b. Malaria falciparum
c. Malaria malariae b. Fecal – oral transmission
d. Malaria ovale c. Fecal – cutaneous transmission
e. Mix infection of malaria vivax and d. Direct contact with skin scales
malaria falciparum e. Intravenous drug abuse

106. Sexual life cycle of malaria parasite take 110. A 10-month-old healthy male infant
place in : travelling with his family to Africa was
a. Human liver cell exposed 4 days ago to his 4-year-old native
b. Human spleen cell African cousin who was ill at the time with
c. Human erythrocyte fever, cough, coryza and conjunctivitis. The
d. Mosquito body baby was at risk to get infected by:
e. Mosquito proboscis a. Corynebacterium diphtheriae.
b. Haemophilus influenzae
107. Plasmodium falciparum is the most c. Bordetella pertussis
dangerous species because : d. Morbilivirus
f. It make long term relaps e. Coxsackievirus
g. Intermittent fever
h. Gametocyte formation take place in 111. Her mother makes an overseas phone
visceral organ call to your office asking your advice.
i. Can make capillary obstruction in Central Appropriate management of this patient
Nervous System by parasite contain includes:
erythrocyte a. Immune globuline
j. There are many drug resistance against P. b. MMR vaccine administration
falciparum c. Measurement of IgG antibody serum
d. Measurement of IgM antibody serum
Question 97-98 are linked to the following case: e. Antibiotics

A 30-years-old woman presents abdominal pain 112. A 3 year old girl came to pediatric
and diarrhea for 3 days duration. She does not outpatient clinic with her parents. Her
complain of nausea, vomiting, or fever. She has no mother complaint that she had watery
sick contacts or significant travel history. A stool diarrhea since 3 days. This was her 3rd
sample is obtained, which reveals larvae form of episode of diarrhea in the last 2 months. Her
of Strongyloides stercoralis. mother also noticed that she was delayed in
growth and development compared to her
108. In condition below adult form of siblings. She hardly gained weight and
Strongyloides stercoralis can be found in height since last year. The physical
whole digestive tract and its larvae can be examination showed that her weight and
found in visceral organ (lung, liver, gall height was below 5th percentile for girls of
bladder) is : her age.
a. Eosinofilia What is the most likely underlying
b. Hyperinfection mechanism that caused her repeated
c. Retrofection episodes of diarrhea?
d. Hypereosinofilia a. Protein catabolism
e. Autoinfection b. Reduced food intake
c. Altered lipid metabolism
d. Reduced bactericidal activities
109. Refer to the case above; further
questioning reveals that the woman frequently 113. A 65-year-old-woman suddenly had
gardens in her backyard. Of the following, paralysis of her left leg. A few days ago, she
which one is the transmitted form? had headache, fever, sore throat, and nausea.
a. Fecal – anal transmission She lives with her son; daughter in law and a
young grandchild who’s just received a B. Platelet count
routine oral vaccination. She is taking
immunosuppressant for her kidney’s C. Dengie igm igg
transplant. Her vital signs and cranial nerve
examination are normal. A head CT scan D. Wbc count
and lumbar MRI are also normal. E. Ht n eritrosit count
What is the most likely transmission of her
infection?
a. Droplet
b. Air borne 3. Worried mother bring her 18month son, rash
c. Fecal-oral develop todayn runny nose, conjunctivitis,
d. Close contact diarrhea, lethargy, irritable. Temp 40c for 3days.
e. Animal bite Diffuse 2-3mm erythemateous rash beginning
114. A 4-year-old child presents with fever, from face. Sever complication?
conjunctivitis, photophobia, posterior
cervical adenopathy, and coryza. Red A. Sinusitis.
lesions with a white center are present on the
B. Oral thrush
buccal mucosa. A generalized erythematous
rash is also noted. What is the most likely C. Osteomyleitis
diagnosis?
a. Rubella D. Encephalitis
b. Kawasaki disease
c. Adenovirus infection E. Otiitis externa
d. Rubeola
e. Varicella
4. 24 yo woman, fever of unknown origin,
persist for 10days, + diarrhea and now become
TROPMED 1
constipated. Brasycardia spleenomegaly
1. 42yo woman ocme to clinic noted white caps neutropenia lymphatocytosis. Most likely sites
in his stools. On stool exam parasit eggs of infection?
contains hexocant were found. Most likely
A. Ileum
source:
B. colon
A. Undercooked pork
C. Appendix
B. Undercooked beef
D. Caecum
C. Ingestion of larvae on the organism
contaminated food E. Duodenum
D. Ingestion of cystic

E. Ingestion of egg 9. 28yo male fever productive cough green


mucous 5 hari lalu elevated BP HR 128 RR 28
temp 39,5 diffuse ronchi di chest x ray,
2. 40 yo man, fever epistaxis nausea since 2days, pneumonia
temp 39. Palpable liver. Rumple leed test+.
A. Sirs b. Sepsis c viremia d bakteremia
Dhf!m confirming lab?
e septic shock
A. Ns1
E. chloramphenicole

10. 27 yo cold. Unprotected sex. How hiv virus


transmitted?
14. Fever high especially at night. Headache and
A saliva b needle stick injury c injecting drugs pain behind eyes. Dignosis
by own needle
A. Severe dengue if he was feeling restless and
d exposure of intact skin w contaminated blood lethargy

e mostly from mother to infant in gestation

15. Irritable, loud noise gara2 pain karena


muscle tightening and back arching. Phisical
11. IgG dan IgM (+) low avidity IgG. examination reveals only a dried pcking on his
Management? umbilical cord, as is the local custom appears
normal until he's stimulated by touch or loud
A no spesific management noise
B antepartum ganciclovir A. Botulism
C fetal usg and amnioncentesis B. tetanus
D CMV immunoglobin C. Meningitis

D. Encephalitis
12. 32 yo jaundice suffesion conjunctivitis E. generalized seizure
trombositopenia hepatosplenomegali...

a HbAs Ag
16. Suka main sm puppy, eosinophilia, enlarged
B liver
C Anti dengue A. Pruritus Ani
D IgM B. cholecystitis
E MAT for leptospirosis C. Billiary cirrhosis

D. Creeping eruption
13. A 25 year old man is admitted to the hospital E. visceral larva migrans
for evaluating increasing fever of unknown
origin along with malaise, headache,
constipation. Nggk vaksin terus ke daerah rural.
Ada bradikardi, stepladder fever, 17. 33 thn cowo with C3 HIV disease.
hepatosplenomegali. Antibiotik yg dikasih apa? Fever,nausea,vomiting,hipotensi. Minum
zidovudin,lamivudine (3TC),abacavir,indinavir.
A. Isnazid Develop fatigue, dyspnea,malaise. Then he stop
medication. PE: hb turun,ast normal,alt
B. vancomycin normal,bilirubin 2 (indirect 1,4). Cause?
C. Clindamycin a. side effect 3TC
D. Metronidazole
b. side effect abacavir 21. Ujungkulon, liver enlarge, on off fever ?

c. side effect indinavir a. Bloodsmear

d. side effect zidovudin b. blood culture

18. 7 thn cowo malaise,anorex,fever,headache. 6 22. Inguinal node swelling, bubo, pustule, ?
classmates kaya gt juga punya symptoms yg diagnosis?
sama. Ada skin lesions macule,papule,vesicle
(multiform). Treatment? a. Tularemia

a. antiviral & antitoxin b. Yaws

b. antiviral & supportive therapy c. Pes

c. supportive therapy & antibiotic

d. supportive therapy without antiviral 23. Muscle pain?

a. Trichinella spiralis

19. 45 thn cowo ada nodul di arm & body pain


with pressure. Erythematous nodules,smooth,
and shiny. Diffuse infiltrate with ill define 24. Leptospirosis tertular dr mana?
border,no erosion. Pain in elbow with nerve
enlargement. Lab?
25. 25 years old, male, Balinese. Generalized
a. slit skin smear seizure. CT scan revealed several calcified
b. skin scrapping regions. Brain biopsy was performed and
revealed scolex with hooklets. Which parasite is
c. KOH the most likely the cause:

d. VDRL A. Echinococcus granulosus

B. Trichinella spiralis

20. 67 thn cewe flu like syndrome. Fever 40 C. Taenia saginata


derajat,anorexia,headache,muscle ache. 3 days
later cough with sputum. Chest x ray D. Taenia solium
parenchymal infiltrate. Specimen untuk pcr? E. Toxocara cati
a. sputum

b. rectal swab 26. 32 years old, male, come to emergency


c. midstream urine departement. 1 Week progressive fever, now
become delirium. Jaundice (+), conjungtival
d. CSF suffusion (+), hepatomegaly (+), Murphy's sign
(-), petechiae on both legs (+). Lab testing was
e. nasopharyngeal aspirate performed:

Leukocyte: 18.000
Thrombocyte: 98.000 D. Blood smear

Creatinine: 4 E. Blood culture

What further examination needed to be done?

A. HBs Ag 30.

B. Widal test

C. Anti-Dengue 31.

D. IgM anti HAV

E. MAT for leptospira 32.

27. 15 years old, male, come to Puskesmas. 33. 10 y/o had bitten by dog 3 days ago in the
Fever since 3 days ago. High fever especially at right foot, he was brought to the clinic and
night. Headache (+), retroorbital pain (+), received antibiotic, analgesic and wound
nausea (+), vomit (+). What is the diagnosis cleansing w/o incision. The bitten site is now
according to WHO 2009? painful, swollen, red and his mother notice that
the boy has fever now. What's possibly happen
A. Severe dengue if he was feeling restless and to the boy?
lethargy
A. Local reaction to trauma and need anti..
B. Dengue fever with warning sign if Torniquet
test (+) B. Abscess right foot must get injection of
antibiotic and wound incision
C. Dengue fever with warning sign if rash (+)
C. Encephalitis stage of rabies infection need to
D. Dengue fever with warning sign if abdominal be isolated and rabies immunoglobulin
pain (+) just once last night
D. Local tetanus infection inject anti tetanus
E. Dengue fever with warning sign if abdominal toxoid and tetanus immunoglobulin
pain (+) more than once last night
E. Rabies infection of right foot inject rabies
immunoglobulin and local vaccination of the
wound
28.

29. Cowo 30 tahun datang ke puskes dengan


keluhan demam, menggigil, sakit kepala sejak 3 34. 40 y/o man complaint of tingling , itching of
hari. Dengan riwayat demam sejak pergi ke legs 30 minutes after swimming in Lindu lake,
ujung kulon national park. Pemeriksaan untuk the next day small papules develops followed by
diagnosa? blisters of legs. Which paracyte infected him?
A. VDRL A. Hymenoptera sp
B. HIV ELISA B. Fasciola hepatica
C. Urine culture C. Fasciolopsis busci
D. Demencator weeks. What is the prevention drug of malaria?
Doxycycline!
E. Schistosoma japonicum

69. a national rescue tem who often travel to


remote areas in indonesia has on-off fever and
chills for 15 days. Arthralgia and headache. PE:
35. 28 y/o man hospitalized for fever more than enlarged liver and spleen. He was brought to ER
7 days, fever higher at noon, felt lower at because suddenly had seizure and finally lost his
morning, fatigue, dizzy, discomfort in stomach, conciousness . what is the characteristic of
constipation, lost appetite. Examination reveals pathogen that might cause the symptom above?
temp 38C, mild pain in abdominal palpated bp
100/70 Hr 70 BPM, tremor, coated tongue. a. invasion of the RBC by parasite
what's the ethiology of the disease?
b. release of parasite antigen to produce
A. Escherichia coli febrile reaction
B. Salmonella typhi c. transmission by injection of
sporozoites
C. Bordetella pertusis
d. digestion of HB in food vacuoles to
D. Shigella dysentriae produce pigment
E. Campylobacter jejuni e. adherence of infected RBC to small
BV endothel

36. 35 y/o man emergency from dyspneu


cyanosis, hemoptisis, chest pain, high fever, 70. fever since 3 days. High fever all day long.
malaise, non productive cough for 1 week. He 3rd day platelet :148.000. HT: 42. Treatment
works as a sheep farmer and was treated for dark suggestion?
black skin lesion. Cbc normal, blood + sputum
negative, hemorrhagic mediastitis. Which a. inpatient hospital and IV fluid drip
virulence factors most likely involved :
b. inpatient puskes with oral fluid intake
A. Exotoxin
c. inpatient hospital with oral fluid
B. Endotoxin intake and IV drip when cant take orally
C. Alpha hemolytic d.outpatient with oral fluid and balik the
next dad u/ cek platelet
D. Lipopolysacharide
e. outparient with oral fluid intake and
E. Antiphagotic factor balik 3 hari lahi u/ cek platelet

37. 45 y.o man, he is going to Mecca for Haji. 71. HT naik 20%. Digigit sama sapa?
What is the vaccine that can be given?
Meningococcal vaccine. A fruit bat

b. anopheles
38. 38 y.o man, he is going to Papua for 2 c. aedes aegepty
d. close contact with infected person 95. a 34 years old man comes to dermatology
with hyperpigmentated macules that spread on
e. direct contact with infected animal his face and his upper arm color lesion white and
feels scaly when t touched. Sometimes he feels
itchy and more itchy when he sweat. Woods
72. multiple papilomatous skin lesion. Wart like light (+). Lab : large, blunt, short hyphae and
srawberry. Best bay diagnosed? thick wall budding spores (spageti and meatball
)
a. tissue culture
a. candida albicans
b. giemsa
b. aspergillus flavus
c. antitoxin detect
c. microsporon canis
d. darkfield (treponema)
d. Malassezia furfur
e. gram stain
e. Trichosporon belgelii
93. a 40 years old male patient came to the
emergency department due to fever and upper
right abdomen pain since 3 days ago. PE : Blood
pressure 120/80 Heart rate 112 RR 28 Temp
39,5. Tenderness right upper quadrat. Lab : 81. 13 old boy, fever, shaking chill, general ill,
WBC 17.000 Hb 13,5 Platelet 100.000 SGOT 75 fatigue muscleached & pain. Playing at garden.
SGPT 17 ureum 80 creatinine 3,3. Hs abdominal Stung by an insect in dorsal lower right arm.
USG shows liver abscess. His condition Skin inflamed, sore, rash, worse in 1st 24hours.
called…. Erythematous surround papulafolicar with pus.
a. sepsis a. Trepanoma pallidum
b. H ducreyi
b. viremia c. M. Leprae
c. bacteremia d. S. Pyogenes
e. C minitussismum
d. severe sepsis

e. anaphylactic schock 82. Rose spot, liver spleen enlarge, drave –


konstipasi, fever 10d, ill, bradikardi. Gold
standard nya?
94. Following test would be the most reliable for
confirming the diagnosis in case above :
83. A patient come with an itchy recured fungal
a. culture of blood infection on the feet and need some advice to be
cured and some prevention measures so the
b. complete blood count diasease will not occure again, doctor already
gave him some medication for fungal infection.
c. nucleic acid based test
What kind of advise can make him healthy?
d. detection of toxin in his serum
f. Don’t eat seafood
g. Use always sandals
h. Keep the skin dry and clean
i. Don’t use shoes all day long
j. Don’t scratch, will spread the infection e) falciparum

1. An-8-year-old girl sustains a large


84. 27 y.o pregnant woman found HIV+ on laceration contaminated with dirt after
prenatal blood testing, CD4T 410, viral load falling from her bike. Her mother can’t
35000. Ga mau antriretroviral. To decrease risk recall how many doses of tetanus toxoid
nyebar ke anak? her daughter has received. Management
of tetanus prophylaxis in this situation
a. Do nothing of unknown history of prior doses of
b. Cesar tetanus toxoid includes:
c. Prophylaxis HSV II A. Adult-type dT
d. Retroviral after delivery B. TIG
e. Breast feed until 6 month C. dT and TIG
D. Haemophilus influenzae type b
conjugate vaccine containing
65. 27 yo man. Snake bite in lower calf 3 hours tetanus toxoid and TIG
ago. Swollen, painful, snake: elapidae. Fang E. DPT
marks +, edema, no blister/necrosis. Wound,
tender + on palpation Answer: C
a) snake toxinspread blood
2. If this patient developed seizures, what
b) venom -> paralytic muscle substance have responsible in this
c) local inflammation response bite = midvenom patient?
A. Tetanospasmin
d) envenomation that cause necrotic tissue -> B. Tetanolisin
pain, edema, abscess formation in right calf C. Adenylate cyclase toxin
D. Fillamentous hemagglutinin
E. Pertactin
Answer: A
66. almond smell (intoxication)
3. A 10-month-old healthy male infant
a) atropine
travelling with his family to Africa was
b) naloxone exposed 4 days ago to his 4-year-old
native African cousin who was ill at the
c) physostigmine time with fever, cough, coryza and
conjunctivitis. The baby was at risk to
d) Na thiosulfate get infected by:
A. Corynebacterium diphtheriae.
B. Haemophilus influenzae
C. Bordetella pertussis
67. fever > 1 week, eritrosit winth ringforms and
D. Morbilivirus
enlarged RBC
E. Coxsackievirus
a) ovale
Answer: D
b) vivax
4. Her mother makes an overseas phone
c) malariae call to your office asking your advice.
Appropriate management of this patient
d)knowlesi includes:
A. Immune globuline 8. In this case all the family members
B. MMR vaccine administration should get the prophylaxis antibiotics as
C. Measurement of IgG antibody follow:
serum A. Amoxicillin 30-50 mg/kg/day p.o.
D. Measurement of IgM antibody for 14 days
serum B. Amoxicillin 30-50 mg/kg/day p.o.
E. Antibiotics for 7 days
Answer: A C. Erithromycin 40-50 mg/kg/day p.o.
for 14 days
5. A 3-year-old boy was brought to D. Erithromycin 40-50 mg/kg/day p.o.
emergency unit Tangerang Hospital for 7 days
with stridor, dyspnea, and “croupy” E. Tetrasiklin 25-50 mg/kg/day p.o. for
cough. 7 days
What is your most likely diagnosis?
A. Bordetella pertussis infection Answer: C
B. Haemophilus influenzae infection
C. Coxsackievirus infection
D. B. Parapertussis infection
E. Corynebacterium diphtheriae
infection

Answer: E

6. What laboratory test to confirm the


diagnosis in this patient?
A. Blood culture
B. Throat swab
C. Chest X-ray
D. Measurement of IgG antibody
serum
E. Complete blood count

Answer: B

7. A 2-year-old boy was brought to


emergency unit Balaraja Hospital with
difficulty in breathing, choking, gasping,
and whoop cough.
What is your most likely diagnosis?
A. Bordetella pertussis infection
B. Haemophilus influenzae infection
C. Coxsackievirus infection
D. Adenovirus infection
E. Corynebacterium diphtheriae
infection
Answer: A

Вам также может понравиться